54
Assessment Exam for USMLE * Step 2: Internal Medicine Answers and Explanations *USMLE is a joint program of the Federation of State Medical Boards of the United States, Inc. and the National Board of Medical Examiners.

Internal Medicine With Answers

Embed Size (px)

Citation preview

Page 1: Internal Medicine With Answers

Assessment Exam forUSMLE* Step 2:

Internal Medicine

Answers and Explanations

*USMLE is a joint program of the Federation of State Medical Boards of the United States, Inc. and the National Board ofMedical Examiners.

Page 2: Internal Medicine With Answers

©2003 Kaplan, Inc.

All rights reserved. No part of this book may be reproduced in any form, by photostat,microfilm, xerography or any other means, or incorporated into any informationretrieval system, electronic or mechanical, without the written permission of Kaplan, Inc.

Not for resale.

Page 3: Internal Medicine With Answers

Internal Medicine Assessment Exam

1

1. A 47-year-old man returns to the physician for follow-upof high blood pressure. On the last visit, the diagnosis ofhypertension was confirmed, and he was started on a lowdose of felodipine. Today he is without complaint anddenies any abnormal weight loss or gain, muscle weak-ness, or skin changes. A full review of systems is unreveal-ing. His past medical history is remarkable for depressionwith sleep disturbance, for which he takes fluoxetine. Hisblood pressure is 165/79 mm Hg, and pulse is 78/min.Physical examination is otherwise unremarkable.Laboratory studies today show:

Serum

Sodium 144 mEq/L

Potassium 2.9 mEq/L

Chloride 105 mEq/L

Bicarbonate 27 mEq/L

Blood urea nitrogen 18 mg/dL

Creatinine 0.9 mg/dL

Calcium 9.1 mg/dL

Albumin 4.1 g/dL

Which of the following is the most appropriate diag-nostic study at this time?

(A) Abdominal CT scan with contrast

(B) Low-dose dexamethasone suppression test

(C) Plasma renin activity and plasma aldosterone con-centration

(D) Renal biopsy

(E) Twenty-four hour urine cortisol

2. A 60-year-old man comes to the clinic for evaluation of aworsening cough. He has a long history of a chroniccough that has bothered him for at least 3 years. Thecough is productive of thick white or yellow sputum andoccurs almost daily. Over the last few days, however, hehas had increased sputum production that is green tingedand low-grade fevers. He has a long history of tobaccouse, having smoked at least a pack a day for the last 40years. Physical examination reveals a large, mildly cyanot-ic appearing man in mild distress. He displays pursed lipbreathing and is using accessory muscles of respirationwith each breath. Chest examination reveals a prolongedexpiratory time, coarse upper respiratory breath sounds,and a mild wheeze on expiration. A review of a recent setof pulmonary function tests, which include spirometry,lung volume, and diffusion capacity measurements,reveals a markedly diminished FEV1 (forced expiratoryvolume in 1 second) of approximately 1 L. Which of thefollowing is an additional finding likely to be seen on pul-monary function testing?

(A) Decrease in the functional reserve capacity

(B) Equivalent decrease in forced vital capacity

(C) Increase in diffusion lung capacity of CO

(D) Increase in total lung capacity and residual volume

(E) Short and narrow flow-volume tracings

Page 4: Internal Medicine With Answers

3. A 30-year-old woman comes to the clinic complainingof an annoying cough that she believes she caught fromher daughter. Apparently, her 3-year-old daughter start-ed attending a new day-care center earlier in the weekand quickly developed a productive cough. The patientreports suffering 2 days of cough productive of thickyellow sputum, low-grade fever, sore throat, andmalaise. Before this episode, she has been healthy. Shedoes not smoke, and, aside from the birth of her daugh-ter, she has not spent a day in the hospital. Vital signsare unremarkable; the patient is afebrile and denies anyfevers or chills at home. Cardiac examination reveals aregular rate and rhythm. Chest auscultation is remark-able for coarse upper-respiratory breath sounds, but nocrackles, egophony, or dullness to percussion. She isbreathing comfortably, without use of accessory mus-cles of respiration, at a rate of 14/min. There is no lym-phadenopathy, pharyngeal exudate, sinus pain, bron-chospasm, or dyspnea. Given this patient’s likely condi-tion, which of the following is the most appropriatenext step in her management?

(A) Chest radiograph

(B) Empiric macrolide antibiotics

(C) No treatment; follow clinically

(D) Sputum culture and Gram stain

(E) Viral serologies and culture

4. A 49-year-old woman comes to your clinic complainingof new-onset rotational vertigo. It began approximately 2months before and occurs several times per day withoutobvious provoking factors. She describes the sensation as“things flying around in circles,” requiring her to sit downor lean against something for balance. She has nauseawith a few episodes of vomiting that accompany thesedizzy spells and has had increased problems with hearinglately. She denies any other changes in her vision, cogni-tion, memory, motor strength, or sensation. She has a pastmedical history of highly symptomatic menopause, anxi-ety, and uterine fibroids. She takes paroxetine, 50 mgonce/day, and estrogen replacement therapy. A Hallpiketest causes no change in her pattern of dizziness. Herhearing is slightly diminished bilaterally but worse on theleft. The rest of her neurologic examination is normal.Over the course of the next several years, this patient mostlikely will experience which of the following?

(A) Complete loss of her vestibular sensation of balance

(B) Complete resolution of her symptoms

(C) A continued increase in the frequency and severityof her vertigo symptoms

(D) Development of refractory tinnitus

(E) Resolution of her vertigo but continued hearing loss

2

USMLE Step 2 Assessment Exam

Page 5: Internal Medicine With Answers

3

5. A 22-year-old African American woman comes to theclinic complaining of fatigue and a rash. Her symptomshave been intermittent and progressive over the last 2years, finally prompting her to seek care. In addition toher rash, which mainly involves the malar portions ofher face and occasionally the area around her ears, shehas suffered from brittle hair, patchy alopecia, diffusearthralgias and myalgias, and intermittent pleuriticchest pain. Her mood has become more labile over theyear, and she feels irritable for no apparent reason.Other than these symptoms, she has been well and hasnot suffered any major illnesses. She currently takes nomedications. Physical examination confirms the sus-pect butterfly rash and reveals two small oral ulcers thatthe patient had not noticed. Laboratory studies areremarkable for a normocytic anemia and a borderlineelevated creatinine. Given this patient’s likely diagnosis,which of the following is the most specific autoanti-body used to diagnose her disease?

(A) Anticentromere antibodies

(B) Antihistone antibody

(C) Antinuclear antibody

(D) Anti-Scl 70 antibody

(E) Anti-Sm antibody

6. A 39-year-old woman comes to the emergency depart-ment complaining of the sudden onset of blanching andpain of her right third digit. It has occurred episodicallyover the past 3 months and is worsening in severity.Episodes come on suddenly, especially during times ofemotional stress and exposure to cold. The blanching lastsfor approximately 30 minutes and is followed by pain inthe affected digit, then cyanosis, then flushing. Betweenepisodes she has no complaints, and none of her otherfingers have been affected. She denies any weight loss,sleep disturbance, fevers, chills, night sweats, or rashes.She takes oral contraceptive pills, naproxen for low backpain, and occasional sumatriptan for migraine headaches.Her past medical history is remarkable only for excessivehemorrhage following the birth of her third child. Thereis no other history of a clotting or bleeding disorder. Sheis in mild distress, but physical examination is normalexcept for her right third digit. It is markedly cyanoticwith a sharp demarcation at the base of the finger. Thereare no skin changes, and it is nontender to palpation.Routine laboratory studies, including a complete bloodcount, electrolytes, creatinine, and urinalysis, are all normal. Which of the following medications is most likely to be effective in controlling this patient’s symptoms?

(A) Amitriptyline

(B) Gabapentin

(C) Nifedipine

(D) Prazosin

(E) Verapamil

Internal Medicine

Page 6: Internal Medicine With Answers

7. A 43-year-old man comes to the clinic with his wife todiscuss a worsening illness. Over the last year, thepatient has suffered progressive neuropsychiatric symp-toms, including severe depression and hallucinations.Additionally, the patient’s wife notes that he hasbecome increasingly bizarre and labile and often seemsunable to control his emotions. The patient wasreferred for cognitive testing, which revealed impair-ment mainly in psychomotor speed, attention, and con-centration, with fairly intact verbal learning and mem-ory. Additionally, over the past few months, the patienthas developed strange jerking motions of his arms thatare involuntary and occur without warning. Before thislast year, the patient had no significant medical history.When asked about his family medical history, thepatient is uncertain as to how his father died, though heknows his father suffered from a severe disease for thelast 10 years of his life. Which of the following is themost likely underlying cause of this patient’s condition?

(A) Abnormal copper regulation, resulting in cerebralcopper deposition

(B) Diffuse dopaminergic neuronal loss in the sub-stantia nigra

(C) Elevated total body iron stores, resulting in diffuseiron deposition

(D) Extracellular deposition of beta-amyloid and neu-rofibrillary tangles

(E) Trinucleotide repeats, resulting in neurotoxic pro-tein expression

8. A 62-year-old alcoholic man is brought to the emer-gency department complaining of nausea. He is brightyellow and obviously drunk, as he smells of alcohol.The patient is now complaining of severe nausea anddull but mild right upper quadrant pain that has per-sisted for the last 2 days. He reports having his lastdrink, a fifth of whiskey, earlier in the morning. Vitalsigns are: temperature 38.0 C (100.4 F), blood pressure100/42 mm Hg, pulse 98/min, and respirations 22/min.Physical examination reveals a thin, jaundiced man inmild distress. Although lung and cardiac examinationare unremarkable, abdominal examination reveals atender, palpable liver 10 cm below the costal margin.There are scattered angiomata across the patient’s chest.Laboratory studies, including a full set of liver functiontests, a complete blood count, a coagulation panel, andserum chemistries, are drawn and currently pending.The patient is planned for admission with a presump-tive diagnosis of alcoholic hepatitis. If this patient isindeed suffering from alcoholic hepatitis, which of thefollowing is an expected pattern of liver function tests?

(A) AST level that is less than the ALT level, with bothgreater than 1000 U/L

(B) AST more than twice the ALT level, ALT level lessthan 500 U/L

(C) Extremely high aminotransferase levels, greaterthan 2000 to 3000 U/L

(D) Near normal alkaline phosphatase and gamma-glutamyltransferase levels

(E) Total protein level that is more than two to threetimes the albumin level

4

USMLE Step 2 Assessment Exam

Page 7: Internal Medicine With Answers

5

9. A 42-year-old woman comes to the clinic requestinganalgesics for pain that began suddenly the day before.She began to feel nauseated and had two episodes ofemesis shortly after the pain began. She describes thepain as wave-like, confined to the right side, and origi-nating in the back. She also notes pain in the groin. Shedenies ever having had pain like this before. Her pastmedical history is remarkable only for three normalpregnancies, and her only medications are calcium sup-plements. Her temperature is 37.0 C (98.6 F), bloodpressure is 145/80 mm Hg, pulse is 108/min, and sheseems acutely distressed. Head, neck, lung, and cardiacexaminations are normal. Abdominal examination isunremarkable and free of tenderness. There is markedtenderness on percussion of the costovertebral angle.Urinalysis shows 2+ blood but is otherwise negative.Laboratory studies show:

Sodium 141 mEq/L

Potassium 4.4 mEq/L

Chloride 108 mEq/L

Bicarbonate 23 mEq/L

Blood urea nitrogen 14 mg/dL

Creatinine 0.8 mg/dL

Calcium 11.9 mg/dL

Albumin 3.8 g/dL

Further questioning and examination would most likelyreveal which of the following?

(A) Chronic diarrhea

(B) Exertional dyspnea

(C) Swollen, painful joints in the fingers and toes

(D) Tetany in arm muscles when blood pressure mea-surement is attempted

(E) Vague periumbilical abdominal pain

10. A 48-year-old man with a long history of diabetes mellitus type 2 is being treated for hyperosmolar non-ketotic coma. Earlier in the day he was found confusedand delirious, sleeping on a park bench. Paramedicsbrought the man to the hospital, where he was found tohave a markedly elevated blood glucose level. Thepatient was started on aggressive fluid rehydration withnormal saline and was started on an insulin drip. Thepatient was admitted to the medicine service for closeobservation and appropriate electrolyte replacement. Inaddition to the patient’s history of diabetes, he suffersfrom chronic hepatitis C, alcoholism, and early diabet-ic nephropathy. Currently, the patient is more alert andis continuing to receive intravenous fluids and insulinand is undergoing regular electrolyte checks. Shortlyafter the patient’s last laboratory draw, a worried nursecalls to let the resident know that the patient’s sodiumlevel is 125 mEq/L and has been since he was trans-ferred to the floor. To interpret this laboratory value,the resident should also ask the nurse for the measuredlevel of which of the following?

(A) Albumin

(B) Alcohol

(C) Creatinine

(D) Glucose

(E) Potassium

11. A 49-year-old man has an episode of colicky left flankpain with radiation to the inner thigh and groin that lastsfor approximately 3 hours and subsides spontaneously.He is advised by his physician to collect and filter all hisurine, in the hope of identifying the urinary stone that heis suspected of having passed. He indeed recovers a 2-mmstone that on further study is determined to be a uric acidstone. An intravenous pyelogram shows a 3-mm fillingdefect present in the left renal pelvis, just above theureteropelvic junction, and a CT scan without contrastshows a bright white spot of the same size on the samelocation. Further management should include which ofthe following?

(A) Acidification of the urine and increased fluidintake

(B) Endoscopic extraction of the stone

(C) Long-term antibiotic therapy

(D) Shock wave lithotripsy

(E) Urinary alkalinization and increased fluid intake

Internal Medicine

Page 8: Internal Medicine With Answers

12. A 39-year-old man with a history of ulcerative colitishas developed crampy right upper quadrant pain overthe past year. The pain has been intermittent and seemssomewhat related to meals. When asked specifically, headmits that recently he has been fatigued and incrediblyitchy, though he has been taking an over-the-counterhistamine blocker to help with the pruritus. He deniesany recent weight loss, change in bowel habits, hema-tochezia, or melena. Examination reveals mildly ictericsclera and mild right upper quadrant tenderness.Laboratory studies reveal:

Liver function tests

Bilirubin, total 2.9 mg/dL

Alkaline phosphatase 259 U/L

AST 52 U/L

ALT 60 U/L

Additionally, a gamma-glutamyltransferase level ismarkedly elevated. Immunologic testing reveals an ele-vated antineutrophil cytoplasmic antibody (ANCA)titer. An abdominal ultrasound reveals some biliarysludge with moderate dilatation of the intra- andextrahepatic bile ducts. Given this patient’s likely con-dition, which of the following is the most likely causeof his recurrent symptoms?

(A) Biliary stricture

(B) Cholesterol gallstones

(C) Colon cancer

(D) Hepatoma

(E) Pigment gallstones

13. A 58-year-old man comes to the clinic complaining ofabdominal and chest discomfort. He has had similarsymptoms for many years that were always relieved byantacids or ranitidine as needed. They begin within 1hour of eating and are exacerbated by spicy foods. Latelyhis symptoms have become steadily worse, and he awak-ens often from sleep with midepigastric burning and anacid taste in his mouth. He denies any weight loss, fevers,chills, or night sweats. His stools have been normal incolor, without any blood. His past medical history is sig-nificant for hypertension, allergic rhinitis, and asthma.He takes atenolol and low-dose aspirin, loratadine, anda corticosteroid inhaler. He uses albuterol inhalers andranitidine as needed. Physical examination is unremark-able, except for mild midepigastric tenderness. Which ofthe following is the most appropriate next step in thispatient’s management?

(A) Empiric therapy with a proton pump inhibitorsuch as pantoprazole

(B) Esophagogastroduodenoscopy (EGD)

(C) Fecal occult blood testing

(D) Serology for Helicobacter pylori

(E) Upper gastrointestinal series

14. A 40-year-old man comes to the clinic complaining offatigue. He also reports mild left upper abdominal andback pain, early satiety, and an unintentional 15-lbweight loss over the past few months. Before this, he hasbeen healthy, never having spent a day in the hospital.His family history is unremarkable; the patient has atwin brother and an older sister, both of whom arehealthy. Physical examination is remarkable forsplenomegaly and mild tenderness over the lower ster-num. A complete blood count shows:

Leukocytes 85,000/mm3

Hematocrit 31%

Platelets 650,000/mm3

A bone marrow biopsy reveals granulocytic hyperplasia.Genetic analysis reveals a translocation between chro-mosomes 9 and 22. Which of the following is the mostappropriate treatment for this patient’s condition?

(A) Cytarabine/busulfan

(B) Hydroxyurea

(C) Interferon-alpha

(D) Interferon-beta

(E) Stem cell transplant

6

USMLE Step 2 Assessment Exam

Page 9: Internal Medicine With Answers

7

15. A 50-year-old man is seen in the emergency departmentcomplaining of abdominal pain and diarrhea. After eval-uation and rehydration, the patient is discharged homewith a diagnosis of viral gastroenteritis. During hisworkup, however, the intern taking care of the patientordered a full array of liver function tests, which revealedan elevated gamma-glutamyl transpeptidase (GGT). Thepatient was advised to follow up with his primary carephysician. The patient now presents in the clinic for eval-uation of this isolated laboratory abnormality. He reportsfeeling healthy since recovering and never having a histo-ry of liver trouble. When asked, he reports drinking onlysocially and denies any illicit drug use. Current medica-tions include acetaminophen for osteoarthritis, atorvas-tatin for dyslipidemia, amiodarone for a persistentsupraventricular tachycardia, and metformin for recentlydiagnosed diabetes. In addition to these conditions, hesuffers from depressive symptoms and was recently firedfrom his job. Physical examination is unremarkableexcept for mild rhinophyma. Which of the following ismost likely to explain this patient’s elevated GGT level?

(A) Acetaminophen

(B) Alcohol

(C) Amiodarone

(D) Atorvastatin

(E) Metformin

16. A concerned nurse pages the resident in the middle ofthe night about a patient who is 1 day post a coronaryartery bypass graft procedure and who is currently inthe surgical step-down unit. Apparently, the patient isconfused, agitated, and having difficulty communicat-ing. The nurse reports that his blood pressure is 95/42mm Hg and pulse is 15/min, according to the cardiacmonitor. He is breathing at 22/min and has a pulse oxy-gen saturation of 95%. Rushing to the patient’s room,the resident finds the patient calm, though somewhatsomnolent, and lying in bed. He is able to communi-cate, albeit slowly, but denies chest pain, shortness ofbreath, or any other symptoms, and just wants to be leftalone. The patient appears diaphoretic and in mild dis-tress. The peripheral pulse is weak and thready, and hisextremities are cool. The cardiac monitor displays thefollowing rhythm:

The nurse asks if a crash cart is needed, and whether acode should be called. Which of the following is themost appropriate first intervention?

(A) Amiodarone bolus

(B) Electrical cardioversion

(C) Epinephrine, intravenous

(D) Lidocaine bolus and drip

(E) Transfer to cardiac care unit

Internal Medicine

Page 10: Internal Medicine With Answers

17. A 52-year-old woman is being evaluated for fatigue andan abnormal gait. She has noticed that over the last yearshe has become increasingly likely to trip and often hasdifficulty maintaining her balance. Further, she feelsexhausted most of the day and tires with minimal exer-tion. She is increasingly forgetful and often has episodeswhere she feels confused and cannot think clearly. Also,her hands and feet occasionally burn and tingle.Physical examination is remarkable for a smoothtongue with mild cheilosis, hyperactive reflexes, and awide-based gait. Laboratory studies reveal a severemacrocytic anemia with a less severe neutropenia andthrombocytopenia. A review of the peripheral bloodsmear shows few reticulocytes and numerous hyperseg-mented neutrophils. Which of the following is the mostappropriate next step to definitively diagnose thispatient’s condition?

(A) Bone marrow biopsy

(B) Ham test

(C) Lumbar puncture

(D) MRI of brain/spine

(E) Schilling test

18. A 58-year-old man comes to the clinic complaining ofshortness of breath. He described the dyspnea as grad-ual in onset over the last few weeks. The patient deniesany pleuritic pain, cough, or orthopnea, and has noprevious cardiac or respiratory medical history. Reviewof systems, however, is concerning for an unexplained20-lb weight loss over the last few months and inter-mittent fevers and night sweats. The patient does notsmoke cigarettes currently, though he has a distant 20pack-year history. He denies using illicit substances andonly rarely drinks alcohol. He has not had any majorillnesses in the past, has had no major operations, andis employed as an accountant. Physical examination isremarkable for reduced breath sounds and dullness topercussion approximately halfway up the righthemithorax. Chest radiograph confirms a right-sidedpleural effusion and some hilar and mediastinal lym-phadenopathy. A diagnostic thoracentesis is performedthat reveals a milky-white, opalescent fluid. Laboratorystudies reveal this fluid to have a markedly elevatedtriglyceride level of 558 mg/dL, a high cell count com-posed mainly of T lymphocytes, and numerous chy-lomicrons. Cytologic examination is unremarkable.Which of the following is the most likely cause of thispatient’s effusion?

(A) Infection with acid-fast organisms

(B) Lymphangioleiomyomatosis (LAM)

(C) Non-Hodgkin lymphoma

(D) Pancreatic adenocarcinoma

(E) Undiagnosed mesothelioma

8

USMLE Step 2 Assessment Exam

Page 11: Internal Medicine With Answers

9

19. A 24-year-old man comes to the clinic complaining of apainful neck. For the past week, he has had an exquisitelytender lump in his neck. Additionally, he has sufferedfrom heart palpitations and insomnia, and he reportsfeeling “grouchy.” Although his past medical history isunremarkable, a review of systems reveals a recent histo-ry of cough, fever, and myalgias, now resolved. Vital signsare: temperature 38.0 C (100.4 F), blood pressure 138/88mm Hg, pulse 121/min, and respirations 20/min.Examination reveals a palpable, non-nodular, andextremely tender thyroid gland. A mild tremor is presentwhen the patient extends his arms. Thyroid function testsshow:

Thyroid-stimulating 0.01 µU/mLhormone (TSH)

Thyroxine (T4) 12.8 µg/dL

A 24-hour radiouptake study performed on this patientmost likely would reveal which of the following?

(A) Diminished radioactive iodine uptake throughoutthe entire thyroid gland

(B) Focal area of increased uptake; rest of the glanddisplaying reduced uptake

(C) Increased uptake of radioactive iodine throughoutthe entire thyroid gland

(D) Nodular areas of increased uptake surrounded byareas of reduced uptake

(E) Normal radioactive iodine uptake scan; no abnor-malities noted

20. A 22-year-old woman comes to the clinic complainingof severe abdominal pain. About 10 days ago, shortlyafter returning from a 2-week vacation in Mexico, shedeveloped fevers and diffuse, crampy, abdominal pain.Additionally, she reports nausea, malaise, and darkurine and that friends have commented more on heryellow-tinged eyes than her dark tan. Given these symp-toms and recent travel history, as well as a markedtransaminitis, the patient is evaluated for a possibleviral hepatitis. A hepatitis screening panel shows:

IgM anti-HAV Positive

IgG anti-HAV Negative

Hep B surface Ag Negative

Hep B surface Ab Positive

IgG anti-Hep C Negative

The patient is treated symptomatically, and symp-tomatically improves over the course of a month. By 3months, her liver enzymes have normalized. Which ofthe following is an additional step in the managementof this patient?

(A) Hepatitis A vaccination

(B) Hepatitis B vaccination

(C) Liver biopsy for evaluation of long-term sequela

(D) Reporting the case to public health officials

(E) Right upper quadrant ultrasound and alpha-fetoprotein level

Internal Medicine

Page 12: Internal Medicine With Answers

21. A 58-year-old woman comes to the emergency depart-ment complaining of chest pain. The pain is substernal,does not radiate, and is worse with inspiration andrecumbency. She is not sure when the pain started butthinks it may have been occurring intermittently forabout 6 hours. Her past medical history is significant fordiabetes mellitus type 2, hypertension, and diabeticnephropathy. Surgical history includes an appendectomyas a child and a recently created atriovenous fistula in theright arm in anticipation of dialysis. She denies tobaccoor alcohol use. Her pulse is 122/min. Multiple “squeaky”sounds are heard on cardiac auscultation. Additionally,the patient has 2+ pitting edema bilaterally. An electro-cardiogram reveals only sinus tachycardia. Laboratorystudies show moderate normocytic anemia, a mildlyincreased troponin T level, mildly elevated potassium,and significantly elevated BUN and creatinine levels.Which of the following is the most appropriate treat-ment for this patient’s chest pain?

(A) Calcium gluconate

(B) Cardiac catheterization

(C) Hemodialysis

(D) Heparin drip

(E) Indomethacin

22. A 40-year-old woman comes to the emergency depart-ment complaining of substernal chest pain. The patientreports having intermittent, nonradiating, dull subster-nal chest pain and pressure that can last up to half anhour. Today, however, her pain seemed worse thanusual, prompting her to come to the emergency depart-ment. Although she denies any shortness of breath,diaphoresis, nausea, or vomiting, she does note that shehas difficulty swallowing during these episodes of chestpain. Her past medical history is significant for stageone hypertension, currently untreated, and mild gas-troesophageal reflux disease (GERD) diagnosed 1 yearago for which she is treated with a proton pumpinhibitor. The patient’s chest pain quickly resolves withtwo sublingual nitroglycerin tablets and she is admittedfor cardiac evaluation and monitoring. Over the next 24hours, she exhibits no electrocardiographic changes orincreases in her cardiac enzymes, though she has somerecurrent chest pain, again relieved with nitroglycerinand a “GI cocktail.” The next morning, the patient has anormal cardiac thallium perfusion scan and is dis-charged with the diagnosis of “atypical chest pain.”Which of the following is an appropriate next step inthe outpatient evaluation of this patient?

(A) Cardiac catheterization and angiography

(B) Esophageal pH probe and manometry

(C) Transthoracic echocardiogram

(D) Trial of H2 blocking agent

(E) Upper endoscopy with gastric biopsy

10

USMLE Step 2 Assessment Exam

Page 13: Internal Medicine With Answers

11

23. A 45-year-old woman is recovering from trans-sphenoidal removal of a pituitary adenoma. Althoughthe surgery apparently went well and there were nointraoperative complications, the patient’s postopera-tive course has been complicated by polyuria, with thepatient losing more than 5 liters of urine a day, cur-rently being replaced with D5 water. Additionally, thepatient’s sodium has been increasing slowly. The inter-nal medicine resident is consulted. A water restrictiontest followed by the administration of intramuscularantidiuretic hormone is performed, and it results in anincrease in urine osmolality. Recent laboratory resultsare as follows:

Pre-Op Post-Op Day 3

Plasma sodium 138 mEq/L 159 mEq/L

Urine osmolality 800 mOsm/Kg 123 mOsm/Kg

Should this patient’s sodium continue to increase, anappropriate intervention would be to administer whichof the following?

(A) Demeclocycline

(B) Desmopressin

(C) Furosemide

(D) Hypertonic saline

(E) Normal saline

24. A 32-year-old woman comes to the clinic complaining ofmalaise, severe pallor, and generalized weakness.Approximately 2 months ago, while working at a ruralhealth clinic in the war-torn Congo, she became extreme-ly ill with a severe gastroenteritis. She was given antibi-otics, though she is not sure which ones, and quicklyrecovered. Since returning to the United States approxi-mately 6 weeks ago, however, she has felt progressivelymore ill. She has never been sick in the past, takes no med-ications, and knows of no diseases that run in her family.A review of symptoms, aside from reduced exercise toler-ance and a mild decrease in appetite, is unremarkable. Shehas suffered no recent fevers, chills, nausea, or diarrhea,and denies any weight loss or night sweats. Vital signs are:temperature 37.0 C (98.6 F), blood pressure 118/70 mmHg, pulse 103/min, and respirations 20/min. Examinationreveals a young, pale woman in no apparent distress.Cardiac auscultation reveals a 2/6 midsystolic murmurheard best at the right upper sternal border that does notradiate. Lungs are clear to auscultation bilaterally, andabdominal examination is benign, without evidence ofhepatosplenomegaly. There are no signs and stigmata ofliver disease or endocarditis. Laboratory studies show:

Hematologic

Hematocrit 17%

Hemoglobin 6.0 g/dL

Leukocytes 3000/mm3 (67% neutrophils,25% lymphocytes,7% monocytes)

Platelets 12,500/mm3

Reticulocytes 0.6%

The rest of the patient’s laboratory studies, including aset of chemistries and liver function tests, are unre-markable. Which of the following is the next mostappropriate diagnostic study?

(A) Blood and stool cultures

(B) Bone marrow biopsy

(C) CT scan of abdomen

(D) Echocardiogram

(E) Erythropoietin level

Internal Medicine

Page 14: Internal Medicine With Answers

25. A 28-year-old woman is being worked up for recurrentvenous clots in her hepatic circulation. Her diagnosishas eluded many physicians, as her symptoms are con-fusing and difficult to relate. In addition to these clots,she has a seemingly intermittent anemia and occasion-ally full pancytopenia, though she always has a reticulo-cytosis. Her anemia has been severe enough to warranttransfusion in the past. Shortly after an appropriatelymatched transfusion, however, she suffered shakingchills and hemoglobinuria. Additionally, she also hasbeen documented to have intermittent hemoglobinuriaand reports occasionally seeing blood in her urine inthe morning. An extensive hypercoagulability workuphas been unrevealing. Multiple reviews of the patient’speripheral blood smear have failed to reveal any mor-phologic abnormalities, and a bone marrow smear,aside from increased cellularity, is unrevealing. Iron lev-els are low, as is serum haptoglobin. Given the patient’sintermittent symptoms and evidence of hemolysis,there is concern that there is a defect in the cell mem-brane stability of the peripheral blood cells. Which ofthe following is an appropriate diagnostic test to orderto evaluate this hypothesis?

(A) Complement levels

(B) Gel electrophoresis

(C) Ham acidification test

(D) Lactate dehydrogenase level

(E) Osmotic fragility test

26. An 18-year-old man comes to clinic complaining offatigue and pallor. He states that he has suffered fromepisodic anemia for as long as he can remember, andnow that he can make his own medical decisions hewould like to have it treated before he leaves for college.Aside from this recurrent anemia, he has had no majorillnesses. The only medication he takes is a daily multi-vitamin. Physical examination confirms a mild pallorand marked splenomegaly. Laboratory studies areremarkable for a reduced hematocrit with an elevatedmean corpuscular hemoglobin concentration (MCHC)and elevated red cell distribution width (RDW). Areview of the blood smear shows numerous small,dense, hyperchromatic erythrocytes that lack centralpallor. A review of the patient’s previous laboratorystudies reveals a positive osmotic fragility test, a lowhaptoglobin level, and an elevated lactate dehydroge-nase level. Which of the following is the most appropri-ate treatment for this patient’s anemia?

(A) Corticosteroids

(B) Erythropoietin

(C) No treatment

(D) Splenectomy

(E) Stem cell transplant

12

USMLE Step 2 Assessment Exam

Page 15: Internal Medicine With Answers

13

27. A 70-year-old man comes to the emergency departmentcomplaining of chest palpitations and light-headed-ness. These symptoms have occurred intermittently foryears but now seem to be occurring almost daily. Thesymptoms usually come and go in a matter of minutes.This morning, however, the symptoms did not resolve,prompting the patient to seek medical care. He has ablood pressure of 132/88 mm Hg, pulse of 170/min,respirations of 22/min, and a temperature of 37 C (98.6F). Examination is remarkable for runs of a normalrhythm interspersed with a regular tachycardia. Areview of the patient’s rhythm strip reveals an alternat-ing normal sinus rhythm and a narrow complex tachy-cardia. Before the patient’s runs of tachycardia, an atri-al premature beat can be seen. Immediately followingthe Q wave is a small upward deflection on the rhythmstrip, which the physician refers to as “a pseudo-Rwave.” On the basis of this information, the patientreceives a carotid massage and is instructed on how toperform a Valsalva maneuver. Despite these interven-tions, his tachycardia continues. Which of the followingis the next appropriate intervention to treat thispatient’s tachycardia?

(A) Adenosine

(B) Amiodarone

(C) Amlodipine

(D) DC cardioversion

(E) Digoxin

28. A 69-year-old woman with a long-standing history ofhypertension reports progressive dyspnea on exertionover the past few weeks to months, as well as not beingable to walk as much as she used to due to “tiring.” Shesleeps with three pillows and often wakens at night feel-ing very short of breath. She has smoked a pack ofcigarettes a days for the past 40 years. She has neverbeen hospitalized. Cardiopulmonary examinationreveals bibasilar rales, an S4, and a weak apical impulsein the sixth intercostal space. There is 2+ pitting edemaof her lower extremities. Which of the following is themost appropriate next step in management?

(A) Admit her to the hospital for an echocardiogramand further treatment

(B) Instruct her to limit her physical activity

(C) Instruct her to limit her salt intake

(D) Prescribe a diuretic and re-evaluate in 3 months

(E) Send her for an echocardiogram and re-evaluate in3 months

29. A 33-year-old man is brought to the emergency depart-ment with altered mental status. The man was foundduring a security check of a large transatlantic cargoship shortly after the boat docked in the United States.Apparently, he is a stowaway who somehow boardedthe ship while it was docked in West Africa to deliverrelief supplies. He is obviously confused and disorient-ed but is able to report boarding the ship to escape fromextreme malnourishment in his country. Currently, heis complaining of diarrhea and nausea. Physical examreveals an anxious, mildly disoriented man with a large,symmetric, hyperpigmented rash on his forearms. Hehas significant temporal wasting and a beefy redtongue. The rest of the examination is unrevealing. Fulllaboratory work is still pending but a complete bloodcount is available and is remarkable only for a mild nor-mocytic anemia. Given this patient’s condition, in addi-tion to caloric support, which of the following is themost appropriate supplement to treat his symptoms?

(A) Niacin

(B) Thiamine

(C) Vitamin B6

(D) Vitamin B12

(E) Vitamin C

30. A pregnant 28-year-old woman comes to the same-dayclinic one morning in late October requesting aninfluenza vaccination. She recently found out she needsto be immunized annually for work because the nursinghome where she is employed requires it of all employ-ees. Although she has had the vaccination in the pastwith no complications, she is concerned that she couldharm her second-trimester fetus. She has no significantpast medical history or comorbidities and has neverhad a particularly severe case of the flu. Her pregnancyhas so far been without complications aside from somemild morning sickness. Which of the following is themost appropriate immunization strategy in this preg-nant patient?

(A) Give the influenza immunization during this visit

(B) No immunization; prescribe as-needed oseltamivir

(C) Provide prophylactic amantadine instead of vaccine

(D) Provide vaccination, but discourage breast-feeding

(E) Recommend immunization immediately post-partum

Internal Medicine

Page 16: Internal Medicine With Answers

31. A 24-year-old man comes to the clinic complaining offlu-like symptoms. For the past 10 days, he has had dif-fuse myalgias, low-grade fevers, headache, and occa-sional nausea and vomiting. He reports being generallyhealthy in the past, though a review of his chart revealsthat he was treated for gonorrhea 4 weeks ago. Exam isremarkable for a diffuse, tender lymphadenopathy aswell as two small, painful mucocutaneous ulcers in hisoropharynx. On further discussion with the patient, hereveals that he has numerous HIV risk factors, includ-ing intravenous drug use and sex with commercial sexworkers. Which of the following is the most appropri-ate HIV test for this patient?

(A) CD4+ T-cell count

(B) ELISA for HIV antibody

(C) HIV RNA level

(D) P24 antigen serology

(E) Western blot

32. A 60-year-old man is being treated for an elevated low-density lipoprotein (LDL) level. Before being started onan HMG CoA reductase inhibitor, he receives a baselineset of liver function tests, which show a markedly ele-vated alkaline phosphatase level. The patient deniesfeeling ill and takes no current medications. His pastmedical history, aside from dyslipidemia, is remarkableonly for a humeral fracture sustained while skiing andfor accelerated presbyacusis. A full review of systemsand physical examination are unremarkable. In tryingto differentiate the source of this patient’s elevated alka-line phosphatase, which of the following is the mostappropriate next test to order?

(A) Alanine aminotransferase

(B) Aspartate aminotransferase

(C) Gamma-glutamyl transpeptidase

(D) Lactate dehydrogenase isoenzymes

(E) Repeat alkaline phosphatase

33. Paramedics rush a 48-year-old man who has suffered awitnessed seizure to the emergency room. The patientwas found in the park, lying on the ground convulsingand drooling, at which time emergency medical helpwas called. He stopped having tonic-clonic movementswhile being transferred to the ambulance. However,during transit he had repeated generalized seizureactivity, which is currently ongoing. In the emergencyroom, the patient is unresponsive and is having repeti-tive movements of his arms, neck, and torso. He iscoughing up blood and appears to have bitten histongue. According to the paramedics’ estimates, thepatient has had some degree of seizure activity for atleast the last 30 minutes. Which of the following is themost appropriate first step in his management?

(A) Diazepam, liquid

(B) EEG monitor placement

(C) Intubate the airway

(D) Phenobarbital infusion

(E) Rectal diazepam

34. A 58-year-old man comes to the emergency room withcrushing substernal chest pain. After an evaluation inthe emergency room, he is admitted to the cardiac careunit with an acute inferior myocardial infarction. A fewhours after admission, his blood pressure is 90/52 mmHg and pulse is 42/min with a normal sinus rhythm.The patient is normally hypertensive and had an admis-sion pulse of 110/min. He is currently pain-free, thoughhe feels a bit light-headed and slow to respond to ques-tions. Which of the following is the most appropriatemanagement?

(A) Emergency CABG

(B) Intravenous atropine

(C) Phenylephrine drip

(D) Transcutaneous pacing

(E) Transvenous pacing

14

USMLE Step 2 Assessment Exam

Page 17: Internal Medicine With Answers

15

35. A 28-year-old African American man comes to theemergency department complaining of urinary tractsymptoms. He is given oral trimethoprim/sulfamethox-azole and told to follow up with his primary care physi-cian. Three days later, he comes to clinic complaining offatigue, shaking chills, and dark-colored urine. He hashad a similar episode once before, as a child, and wastold he had a food allergy to beans. Physical examina-tion, aside from some mild pallor, is unremarkable.Laboratory studies show:

Hematologic

Hemoglobin 7.5 g/dL

Hematocrit 24%

Leukocytes 4000/mm3

Platelets 143,000/mm3

Reticulocytes 7%

A peripheral blood smear is remarkable for polychro-masia and numerous acanthocytes. Which of the fol-lowing is the most likely underlying cause of thispatient’s condition?

(A) Abnormal cellular membrane proteins

(B) Drug-induced autoantibody formation

(C) Erythrocyte enzymatic deficiency

(D) Hapten-mediated cellular destruction

(E) Inherited hemoglobinopathy

36. A 55-year-old man with a long smoking history comesto the clinic for evaluation of a chronic cough. Thecough has bothered him for at least 3 years now. It isproductive of thick white or yellow sputum and occurson most days. He sheepishly admits that he has a longhistory of tobacco use, having smoked at least a pack aday for the last 40 years. Physical examination revealssome mild wheezes on expiration and coarse upper res-piratory breath sounds. Pulse oximetry is remarkable foran oxygen saturation of 93%, which drops to 91% withexercise. Pulmonary function tests reveal a decreasedforced expiratory volume in 1 second with a relativelypreserved forced vital capacity, increased lung volumes,and a moderately reduced diffusion lung capacity of CO.In addition to smoking cessation, which of the followingis the most effective and appropriate long-term out-patient medical treatment for this patient?

(A) Albuterol

(B) Flunisolide

(C) Ipratropium

(D) Oxygen therapy

(E) Prednisone

37. A 25-year-old woman comes to the clinic complainingof pain and swelling in the right elbow. She noticed theswelling 2 days before presentation. She reports that shehas been feeling somewhat feverish for the last severaldays and also reports nonspecific symptoms such asarthritis and myalgia. She denies any other medical his-tory. She is not taking any medications. She denies anydrug use. Her temperature is 38.3 C (101.0 F), bloodpressure is 120/80 mm Hg, and pulse is 100/min. Shehas a vesiculopustular skin lesion on her abdomen anda mildly tender and swollen right elbow. Which of thefollowing is appropriate pharmacotherapy?

(A) Acyclovir

(B) Azithromycin

(C) Ceftriaxone

(D) Ciprofloxacin

(E) Vancomycin

Internal Medicine

Page 18: Internal Medicine With Answers

38. A 42-year-old woman received a lung transplant 5 months ago. She is on several immunosuppressantsfor her transplant. She now complains of several days offever, chills, shakes, cough, and hemoptysis. She reportsno recent long-distance travel for immobilization. Shehas no leg swelling or palpitation. A left upper lobe cav-itary lesion is seen on chest x-ray. Induced sputum isobtained and returns positive for acid-fast bacilli.Laboratory studies are otherwise unremarkable.Mycobacterium tuberculosis infection is suspected.Which of the following is the most appropriate man-agement of this patient?

(A) Isoniazid

(B) Isoniazid and rifampin

(C) Isoniazid, rifampin, and pyrazinamide

(D) Isoniazid, rifampin, pyrazinamide, and ethambutol

(E) Rifampin

39. A 63-year-old man is admitted to the hospital for man-agement of pneumococcal pneumonia. He has been pre-viously healthy until he developed fevers and a coughproductive of brownish sputum. He is admitted foradministration of intravenous antibiotics. Over the nextfew days, he is noted to be progressively lethargic andconfused. Evaluation for meningitis, including a lumbarpuncture, is negative. A CT scan of the head is negative.Chest x-ray reveals a persistent pneumonia in the rightlower lobe. His serum sodium is 125 mEq/L. Blood cul-tures and remaining laboratory studies are normal. Hisjugular venous pressure is 9 cm H2O. Examinationreveals crackles in the right lower lung field. He has nopedal edema. The patient has been eating fairly well andhas not been noted to be aspirating. His fractional excre-tion of sodium (FeNa) is 0.5%. Which of the followingis the most appropriate management?

(A) Administer a bolus of normal saline

(B) Administer furosemide

(C) Administer salt tablets

(D) Expand antibiotic coverage

(E) Restrict fluid to 1 L of water daily

40. A 30-year-old man comes to the emergency departmentcomplaining of chest palpitations. For the last few hourshe has been light-headed and feels as if his heart is “burst-ing out” of his chest. He has had some similar fleetingepisodes of palpitations in the past, but because they havebeen transient, he has simply attributed them to stressand coffee and has not sought emergency medical treat-ment. When checking the radial pulse, the physician notesan irregular, rapid heart rate of approximately 130/min.The rest of the patient’s vital signs are stable. An electro-cardiogram shows atrial fibrillation with a ventricular rateas high as 230/min. The QRS complexes appear widened.An old electrocardiogram obtained 1 year ago as part of apre-employment physical examination is available. Itreveals a short PR interval with a strange up-sloping ofthe R wave. Which of the following is the most appropri-ate treatment for this patient’s arrhythmia?

(A) Digoxin

(B) Emergent electrical cardioversion

(C) Intravenous diltiazem

(D) Metoprolol

(E) Procainamide

41. A 21-year-old college student comes to the clinic con-cerned about a new tremor and increased difficultyconcentrating. He has a history of behavioral disorderssince high school and a previous diagnosis of attentiondeficit hyperactivity disorder (ADHD), but recentlythings have become much worse. Over the past year hehas developed a tremor in both arms and difficulty con-centrating. He has felt increasingly distant from friends,who have found his behavior to be odd and different,though he himself has noticed no change, aside fromperhaps some mild irritability. A review of symptomsreveals some mild dysphagia, episodic dysarthria, andevidence of mild depression. His past medical history,aside from the ADHD, is significant for autoimmuneliver disease treated with steroids. Ocular examinationis remarkable for a dark greenish-brown deposit sur-rounding the iris. Abdominal palpation reveals markedhepatosplenomegaly; neurologic exam shows a mildtremor, dysmetria, and an ataxic gait. Which of the fol-lowing is an expected laboratory abnormality in thispatient that can be used to diagnose his disease?

(A) Elevated serum copper concentration

(B) Elevated urinary copper excretion

(C) Increased circulating serum ceruloplasmin

(D) Low amount of circulating free copper

(E) Reduced hepatic copper concentration

16

USMLE Step 2 Assessment Exam

Page 19: Internal Medicine With Answers

17

42. A 66-year-old obese woman has had deep, aching pain inher hands and right knee for the past 4 years. The painoccurs with use of the joint and is relieved with rest andcessation of weight bearing. She has no other symptomsand does not take any medications. Her temperature is 37 C (98.6 F). Physical examination shows joint margintenderness and fine crepitations and limitations to jointmotion. There is bony swelling of the proximal and distalinterphalangeal joints. The remainder of the examinationis unremarkable. In addition to weight loss and exercise,which of the following is the most appropriate initialtreatment?

(A) Acetaminophen

(B) Joint arthroplasty

(C) Methotrexate

(D) Prednisone

(E) No additional therapy is indicated

43. A 58-year-old man comes to the clinic for a health carecheckup. He wishes to discuss his risk for having amyocardial infarction in the next 10 years, as his brother,who is 48 years old, recently suffered from a near fatalone. He denies any chest pain, shortness of breath,orthopnea, or other cardiac symptoms, though he admitshe has been ill in the past. Past medical history is remark-able for diabetes mellitus, diagnosed 8 years ago, and astroke 2 years ago. He recently quit smoking after a 40pack-year history and is mildly overweight. Physicalexamination is unremarkable, though an electrocardio-gram shows high voltages in the precordial leads. Giventhis patient’s risk factors, which of the following is themost appropriate strategy for cardiac risk stratificationand to determine treatment goals?

(A) Coronary computed tomography

(B) Electrocardiographic exercise testing

(C) Exercise-stress echocardiography

(D) Myocardial perfusion scintigraphy

(E) No further testing necessary

44. A 68-year-old man is recovering from a case ofcommunity-acquired pneumonia. He now returns tothe clinic complaining of continued malaise, fevers, andchills, though his cough has improved somewhat. Hehas finished a 10-day course of levofloxacin, as pre-scribed, and feels it has helped him somewhat, thoughhe is still feeling ill. Physical examination is remarkablefor a low-grade fever and for reduced lung sounds anddullness to percussion one third of the way up the righthemithorax. A chest radiograph confirms the suspectedeffusion, and a diagnostic thoracentesis is performed,revealing a thick, exudative-appearing fluid. The effu-sion occupies approximately one third of the righthemithorax and partially layers on right lateral decubi-tus radiographs. There is some difficulty draining thefluid, but approximately 50 mL of the effusion isremoved. Laboratory studies show:

Pleural fluid

Glucose 15 mg/dL

pH 7.04

Cell count 10,000 cells/mL, mainly PMNs

Gram stain Numerous gram-positive cocci

Given this patient’s presentation, which of the followingis the most appropriate management?

(A) Admit for broad-spectrum antibiotics pendingculture results

(B) Admit patient to hospital for chest tube drainageand antibiotics

(C) Prescribe oral ciprofloxacin; follow serial radio-graphs

(D) Refer patient to cardiothoracic surgery for openpleurectomy

(E) Repeat radiograph and perform repeat thoracentesisas necessary

Internal Medicine

Page 20: Internal Medicine With Answers

45. A 34-year-old man with a known diagnosis ofHIV/AIDS is brought to the emergency department following a witnessed seizure. Although he is still mildly confused in the emergency department, hereports a history of weakness, fatigue, chronic diarrhea,and, over the last week, progressive “heaviness” andweakness of the left side of his body. Reviewing hischart, the physician sees that his last CD4+ lymphocytecount was 148 cells/mm3, and the HIV viral load was400,000 copies/mm3. Physical examination reveals athin, cachectic-appearing man with a mild left hemi-paresis. MRI reveals three ring-enhancing lesions,located in the right basal ganglia, the right frontal lobe,and the left parietal lobe. Which of the following is themost appropriate first treatment for this patient?

(A) Amphotericin B

(B) Glucocorticoids

(C) Pyrimethamine/sulfadiazine/leucovorin

(D) Surgical resection

(E) Trimethoprim/sulfamethoxazole

46. A 58-year-old woman comes to the clinic for a routinemedical checkup. She has no major medical problemsand denies any recent illness aside from a couple of“colds” and an annoying, persistent cough. She does nothave a history of asthma and has never had pneumonia.On review of systems, the patient reports feeling morefatigued, which she attributes to aging, though she deniesany weight loss or other constitutional symptoms. Shedoes not exercise regularly and has smoked one to oneand a half packs of cigarettes per day since age 20 years.Her cough has bothered her for the last few years andseems to be getting worse. She reports producing approx-imately 1-2 tablespoons of white or yellow tinged spu-tum. Physical examination reveals a prolonged expiratoryphase of respiration, scattered rhonchi that clear withcoughing, and a faint end-expiratory wheeze.

Spirometry shows:

Measurement Percent ofPredicted

Forced vital capacity (FVC) 92%

Forced expiratory volume in 42%1 second (FEV1)

Ratio of FEV1 to FVC 48%

Forced expiratory flow, 28%midexpiratory (FEF25-75%)

Given this patient’s presentation, which of the follow-ing is most likely to improve her condition over the nextfew years?

(A) Aggressive use of inhaled beta-agonists

(B) Daily use of inhaled glucocorticoids

(C) Intensive efforts aimed at smoking cessation

(D) Lung reduction surgery or transplantation

(E) Regular use of inhaled cholinergic antagonists

18

USMLE Step 2 Assessment Exam

Page 21: Internal Medicine With Answers

19

47. A 33-year-old woman comes to the clinic complainingof fluctuating weakness and fatigability. She often getstired walking up stairs or has difficulty rising from achair. Additionally, she occasionally suffers from doublevision and recently has begun to have episodes ofslurred speech and difficulty speaking. Her symptomsusually improve with rest or sleep. These problems haveplagued her for the last 2 years, but recently these“spells” seem to have been longer and more severe,though she currently feels “okay.” Review of systems isunremarkable; she denies any chest pain, orthopnea,dyspnea, muscle pain, seizure-like activity, weightchanges, or other constitutional symptoms. Family andsocial history are likewise unremarkable; there are noneurologic diseases in the family and the patient hasnever smoked cigarettes. Physical examination revealsmild, bilateral ptosis, weakness of upward gaze, andmild proximal muscle weakness. With progressive test-ing, the patient’s upward gaze and hand strengthbecome weaker. Based on the patient’s symptoms, a testdose of edrophonium is administered to elucidate thecause of her weakness. After receiving a total dose of 10mg of edrophonium, which of the following is a likelyresponse in this patient?

(A) Gradual improvement in ptosis, diplopia, anddysarthria

(B) No immediate effect from the first dose of edro-phonium

(C) Reproduction of severe symptoms of weakness

(D) Temporary increase in pulse and sympathetic tone

(E) Transient increase in the strength of her handgrip

48. A 45-year-old man comes to clinic complaining offatigue, palpitations, and dizziness made worse bystanding. A focused history reveals that the patient hashad recent episodes of melena, though he denies anyhematochezia or hematemesis. A stat blood count con-firms the suspected anemia, revealing a hematocrit of25%. Although the patient does not have any clinicalevidence of liver disease, he does have a 10-year historyof hepatitis C. Although he has had normal liver func-tion tests, a recent right upper quadrant ultrasoundrevealed an irregular, coarse-appearing liver and anenlarged spleen. Given this evidence of cirrhosis andportal hypertension, there is concern that the patientmight have variceal bleeding. He is admitted for bloodtransfusion and an upper endoscopy. Endoscopy fails toreveal any evidence of esophageal or significant gastricvarices. The gastric mucosa does, however, appearabnormal. In some areas a fine white reticular pattern isinterspersed between pink, moist mucosa, creating a“snakeskin” appearance. Other areas, particularly in thebody of the stomach, reveal increased vascularity, smallareas of oozing, and multiple, punctate, subepithelialhemorrhages. The patient is given a tentative diagnosisof portal hypertensive (congestive) gastropathy as thecause of his bleeding. He is observed over the next 2days and is found to have a stable hematocrit. Which ofthe following is an additional appropriate treatment forthis patient?

(A) Beta-blocker therapy

(B) Histamine-H2 blocker

(C) No additional treatment

(D) Proton pump inhibitor

(E) Surgical resection

Internal Medicine

Page 22: Internal Medicine With Answers

49. A 28-year-old man with a history of hereditary sphero-cytosis comes to the clinic for a routine physical exam-ination. He reports being in good health and has nocurrent complaints, though he needs a health examina-tion form filled out for insurance purposes. Whenasked about his history of hereditary spherocytosis, hereports having the disease as a child but having noproblems since he was probably 8 years old, when hewas treated. The patient currently takes no medications,and a review of systems is unremarkable. Physicalexamination reveals a well-healed surgical scar in theleft upper quadrant but is otherwise unremarkable.Laboratory studies include a complete blood countwith an elevated mean corpuscle hemoglobin concen-tration but a normal hemoglobin level. A review of theperipheral blood smear reveals numerous spherocytesand aggregates of denatured hemoglobin in the redblood cell periphery. Which of the following is anappropriate treatment for this patient?

(A) Corticosteroids

(B) Dental antibiotic prophylaxis

(C) Folic acid supplements

(D) No additional treatment

(E) Pneumococcal vaccine

50. An 84-year-old woman is brought from her assisted living facility to the urgent care clinic. She is forgetful andsomewhat confused but is able to convey to you that sheis having severe muscle cramps and generalized weak-ness. The symptoms have lasted at least a week, are worsein her lower extremities, and are unrelieved with rest. Sheis a poor historian but does state that she started a newmedication for her high blood pressure 1 to 2 monthsago. Unfortunately, she cannot remember the name ofher high blood pressure medication, and the assisted liv-ing facility did not send a list of the patient’s medications.Assuming her current symptoms of weakness, legcramps, and confusion are caused by her new medica-tion, which of the following is the most likely culprit?

(A) Captopril

(B) Clonidine

(C) Felodipine

(D) Hydrochlorothiazide

(E) Metoprolol

51. A 68-year-old man comes to the clinic complaining ofsevere dyspnea on exertion. He also reports a chroniccough that is worse at night and that produces a smallamount of whitish sputum. The symptoms have beenprogressive since his retirement 2 years ago from thefamily business, a large marble and quartz quarry. Hedenies any associated symptoms; he has had no hemoptysis, fevers, wheeze, or pleuritic pain. Past med-ical history is unremarkable, but the patient does admitto a 40 pack-year history of tobacco use. A review ofsystems is unrevealing. Physical examination is remark-able for dry inspiratory crackles bilaterally and club-bing of the fingers. A chest radiograph shows multiple3- to 5-mm nodules, with a greater number of nodulesin the upper lobes. Two larger, ill-defined masses arepresent in both the left and right upper lobes, thoughno obvious cavitation is seen. Pulmonary function test-ing reveals a severe ventilatory defect and a restrictivelung disease pattern. Shortly after this clinic visit, thepatient dies from hypoxic respiratory failure. At therequest of the family, an autopsy is performed. Giventhis patient’s presentation, which of the following is alikely finding on autopsy?

(A) Evidence of chronic, postobstructive necrotizingpneumonia

(B) Destruction and incomplete replacement of lungparenchyma and massive fibrosis

(C) Masses composed of acid-fast organisms andnecrotic tissue

(D) Pathologic evidence of widely metastatic small celllung cancer

(E) Pleural-based tumors with encroachment intolung parenchyma

20

USMLE Step 2 Assessment Exam

Page 23: Internal Medicine With Answers

21

52. A 58-year-old man comes to the clinic complaining of arash on his legs that has developed over the past fewweeks. The rash is itchy and painful and seems recentlyto have had some blistering. Additionally, he reports ahistory of severe arthralgias and near-debilitatingfatigue. When asked about his past medical history, hetells you that he has been diagnosed with “some type ofhepatitis.” Physical examination is remarkable for a palpable, purpuric rash over both lower extremities andhepatosplenomegaly. Routine laboratory studies show:

Blood, plasma, serum

Sodium 136 mEq/L

Potassium 3.8 mEq/L

Chloride 111 mEq/L

CO2 24 mEq/L

Urea nitrogen 29 mg/dL

Creatinine 2.1 mg/dL

Glucose 109 mg/dL

The patient’s creatinine level measured 1 year ago was1.2 mg/dL. Given this patient’s presentation, which ofthe following is the most likely underlying condition?

(A) Hepatitis A

(B) Hepatitis B

(C) Hepatitis C

(D) Hepatitis D

(E) Hepatitis E

53. A 40-year-old woman comes to the clinic complainingof bleeding from her gums and recurrent nosebleeds.These symptoms have been occurring off and on for thelast few months. She has never had trouble like this inthe past. Indeed, 1 year ago she had an elective chole-cystectomy with no postoperative bleeding. She hasbeen healthy all her life; she has had no prior hospital-izations and takes no medications. Physical exam, asidefrom mild gingival hypertrophy and scattered oralpetechiae, is unremarkable. Laboratory studies show:

Bleeding time 6.2 min

Hemoglobin 13.0 g/dL

Hematocrit 39.5%

Leukocytes 7000/mm3

Platelets 12,000/mm3

Partial thromboplastin 22 sectime

Prothrombin time 10 sec

A review of the blood smear reveals reduced plateletswith slightly increased volume but no other abnormal-ities or platelet clumps. With the exception of herplatelet count, the patient’s lab work is essentiallyunchanged from preoperative labs taken 1 year ago.Which of the following is the most appropriate nextstep in management?

(A) Measure platelet autoantibodies

(B) Perform a bone marrow biopsy

(C) Provide a trial of oral corticosteroids

(D) Refer to surgery for splenectomy

(E) Start intravenous immunoglobulin

Internal Medicine

Page 24: Internal Medicine With Answers

54. A 39-year-old man comes to the clinic for a routineannual physical examination. He has no current com-plaints and currently feels well. However, he mentionsthat a few weeks ago he passed out while on an airplane.He was on his way back from Mexico, where he hadpicked up a bad case of gastroenteritis. While waiting touse the lavatory, he suddenly blacked out and wasapparently unconscious for 10 to 20 seconds, accordingto the flight attendant. He felt a little dizzy after theevent and was mildly nauseous but in general reportsbeing alert and oriented once he recovered conscious-ness. He refused to go the emergency room once theplane landed, and has had no recurrent episodes of syn-cope or gastroenteritis. He denies any history of suchevents, or of any seizures. A full review of systems, withemphasis on cardiac and neurologic symptoms, is unre-markable, as is the physical exam. An electrocardiogramis unremarkable. Which of the following is the mostappropriate test to order for this patient’s syncope?

(A) Exercise stress test

(B) Head CT scan

(C) No additional testing

(D) Signal-averaged electrocardiogram

(E) Upright tilt testing

55. A concerned daughter brings her 89-year-old father toclinic for a checkup. His wife died about a year ago, andsince then he has lived alone. He has been understand-ably grieved and reports having a diminished appetite.He cooks for himself and tries to stay involved in thelocal senior center. Over the past few months, however,he has suffered some disturbing skin changes. On hislegs, numerous ecchymotic patches are evident. On closeexamination of his skin, hemorrhagic areas surroundthe hair follicles and the hairs themselves appear frag-mented. Splinter hemorrhages are present in his nailbeds, and the gums surrounding his few remaining teethappear inflamed and mildly hemorrhagic. Coagulationstudies are ordered but are unremarkable; the pro-thrombin time and partial thromboplastin times arewithin normal limits. A complete blood count, however,is remarkable for a hematocrit of 29% with normocyticindices. Which of the following is the most appropriatesupplement to treat this patient’s symptoms?

(A) Niacin

(B) Vitamin B6

(C) Vitamin B12

(D) Vitamin C

(E) Vitamin K

56. A 45-year-old man is brought to the clinic with increas-ing confusion and difficulty ambulating. His deteriora-tion has been pronounced over the past 2 years. He hasno known medical issues and is not taking any medica-tions. He has no allergies. On examination, he is notedto have pupillary accommodation but no reaction. Histreponemal serology is positive, and his RPR titer ispositive in the blood and the cerebrospinal fluid. Otherlaboratory studies are unremarkable. An MRI of hisbrain is unremarkable. Which of the following is themost appropriate treatment?

(A) Ceftriaxone, 2 g daily for 2 weeks

(B) Doxycycline, 100 mg twice daily for 7 days

(C) Penicillin benzathine, 2.4 million units every weekfor 3 weeks

(D) Penicillin G, 3 million units every 4 hours for 1week

(E) Penicillin G, 3 million units every 4 hours for 2weeks

57. A 45-year-old woman is admitted to the hospital for further evaluation of progressive shortness of breathwith minimal exertion. The symptoms have been ongo-ing for the last year. The patient denies cough, fever,chills, palpitations, chest pain, or exposure to chemicals.She is moderately obese and has been trying to loseweight. She has no other medical issues and is not tak-ing any medications. She has no drug allergies. Her hus-band reports that she has been snoring intermittently atnight. Her vital signs are within acceptable limits. Lungsare clear. Heart rate and rhythm are normal. Anechocardiogram reveals an elevated right ventricularsystolic pressure, raising the issue of pulmonary hyper-tension. An electrocardiogram is unremarkable. All lab-oratory studies are within normal limits. Which of thefollowing is the most appropriate next step in the man-agement?

(A) Exercise stress test

(B) Left heart cardiac catheterization

(C) Lung biopsy

(D) Polysomnography test

(E) Right heart cardiac catheterization

22

USMLE Step 2 Assessment Exam

Page 25: Internal Medicine With Answers

23

58. A 76-year-old man comes to the emergency departmentcomplaining of lower back pain. He is an ex-smokerand reports that he has a history of prostate cancertreated in another state 5 years ago. He is currently onno medications. Although the pain started out mild, hereports that it has been getting progressively worse overthe last few days. Arising from a seated position makesthe pain worse, and it has repeatedly woken him fromsleep. He cannot seem to find a position that alleviatesthe pain. He also states that twice in the last 24 hours hehas lost control of his bladder. Physical examinationreveals a well-nourished elderly man in no apparentdistress, but he has point tenderness in the upper lum-bar region of his spine. Neurologic examination revealssome loss of motor strength in all muscle groups inboth lower extremities. Plain spine films taken onarrival are read as unremarkable. Which of the follow-ing is the most appropriate next diagnostic study?

(A) Bone scan

(B) CT scan of the brain

(C) Lumbar puncture

(D) MRI

(E) Serum PSA

59. A 25-year-old woman comes to the emergency depart-ment because of acute shortness of breath that started 4hours before admission. She denies recent chest pain ordyspnea on exertion. She has no prior medical history.Her medications include ethinyl estradiol/levonorgestreldaily for birth control. She has a 10-year history of smok-ing a pack of cigarettes daily. Physical examination isnotable for an anxious woman. Her blood pressure is135/80 mm Hg, pulse is 110/min and respirations are34/min. Lungs are clear. A ventilation-perfusion scan is atintermediate probability for a pulmonary embolus.Which of the following is the most appropriate manage-ment at this time?

(A) Administer aspirin

(B) Administer heparin

(C) Administer streptokinase

(D) Administer warfarin

(E) Place an inferior vena cava filter

60. A 58-year-old woman is rushed to the emergencydepartment after an overdose of pills. In an apparentsuicide attempt, she swallowed handfuls of pills 2 hoursago. Judging from bottles found at her bedside, the pillsincluded digoxin, aspirin, acetaminophen, oxycodone,and amitriptyline. Further, it seems she washed downthe pills with a hefty dose of alcohol, as two empty bot-tles of vodka also were found. The patient is currentlystuporous and difficult to arouse. A nasogastric tube isplaced and the stomach is lavaged. Activated charcoal,followed by further lavage, then is started. In using acti-vated charcoal in this patient, it is important to realizethat charcoal does not affect the absorption of which ofthe following?

(A) Acetaminophen

(B) Alcohol

(C) Amitriptyline

(D) Aspirin

(E) Digoxin

61. A 32-year-old man comes to clinic for a routine healthcheck. He wants to have his vision checked because hethinks he is going blind. As an example, he says thattwice in the last 6 months he has had car accidents driv-ing home late from work. In fact, as he thinks about it,his vision seems worse at night, though he is convincedhis vision is diminished in the daytime as well. His pastmedical history is significant for poorly controlledCrohn disease and an appendectomy 4 years ago. Hetakes sulfasalazine and is currently on a prednisonetaper. Vital signs are within normal limits, and visualexamination reveals a visual acuity of 20/30 OD and20/20 OS. Examination of his conjunctivae reveals areasof keratinization and debris. His skin also shows areasof hyperkeratosis and seborrhea. Which of the follow-ing is the most likely cause of this patient’s symptoms?

(A) Vitamin A deficiency

(B) Vitamin B deficiency

(C) Vitamin D deficiency

(D) Vitamin E deficiency

(E) Vitamin K deficiency

Internal Medicine

Page 26: Internal Medicine With Answers

62. A 54-year-old woman develops left flank pain ofsudden onset with radiation to her inner thigh andlabia. The pain is colicky and is accompanied by irrita-tive voiding symptoms. Physical examination shows apatient in moderate distress, thrashing about on thestretcher, but who is otherwise noncontributory.Urinalysis shows microhematuria, and an emergencyCT scan reveals the presence of a 4-mm calcified stoneat the lower end of the left ureter, just above the junc-tion with the bladder. She is given pain medication andis placed on intravenous fluids on the expectation thatshe will pass the stone. Three hours later she developschills, a fever spike to 40.3 C (104.5 F), left costoverte-bral angle tenderness, nausea, and vomiting. A repeaturinalysis shows infected urine. Further managementshould include which of the following?

(A) Addition of antibiotics to the current expectanttherapy

(B) Alkalinization of the urine and increased fluidintake

(C) Emergency endoscopic removal of the stone

(D) Emergency nephrostomy and intravenous anti-biotics

(E) Shock wave lithotripsy

63. A 73-year-old man comes to the clinic for medical clear-ance before an elective surgical repair of a 6-cm abdomi-nal aortic aneurysm. His cardiovascular history indicatesthat 8 months ago he suffered a transmural myocardialinfarction (MI), from which he has recovered completely.He also gives a history of progressive, unstable, disablingangina that is no longer responding to medical manage-ment. Physical examination is noncontributory, and hespecifically has no signs of congestive heart failure. Hiscurrent electrocardiogram (taken at rest) shows evidenceof his former infarction but is otherwise nondiagnostic.Before the proposed surgery is undertaken, it would bewise to do which of the following?

(A) An echocardiogram

(B) An exercise tolerance test and thallium scan

(C) Perform cardiac catheterization and coronaryangiogram

(D) Treat him with diuretics and digitalis

(E) Wait until 1 year has elapsed since his MI

64. A 28-year-old woman has had difficulty climbing stairsover the past few days. She also notes tingling in herarms and legs. She has no chronic medical conditions;however, she had a “cold” 2 weeks earlier. Physicalexamination shows bilateral lower extremity weaknessand mild weakness in the arms, intact sensation, and nodeep tendon reflexes. Lumbar puncture shows elevatedprotein without an associated increase in cell count.Which of the following is the most appropriate initialtreatment?

(A) Glucocorticoids

(B) Interferon-beta

(C) Plasmapheresis

(D) Pyridostigmine

(E) Thymectomy

65. A 59-year-old woman comes to the clinic with chestpain over the past few hours that is different from hertypical angina because it is not relieved by rest. She hasa history of diabetes mellitus and untreated hyper-cholesterolemia. When the clinic physician reaches herroom and examines her, she is pain free and is tellingthe nurse about a new exercise routine with a personaltrainer that she began earlier in the week. Her bloodpressure is 160/90 mm Hg, pulse is 85/min, and respi-rations are 22/min. Examination reveals that she isdiaphoretic and has an S3 gallop and bibasilar coarserales. A chest radiograph shows mild pulmonaryedema. An electrocardiogram taken on arrival to theclinic shows ST segment depressions in leads V3, V4,V5, and V6. Which of the following is the most likelydiagnosis?

(A) Costochondritis

(B) Musculoskeletal pain

(C) Myocardial infarction

(D) Myocardial ischemia

(E) Pulmonary embolism

24

USMLE Step 2 Assessment Exam

Page 27: Internal Medicine With Answers

25

66. A 58-year-old man has fatigue and occasional dizzinessfor the past 3 months. He has been told by family andfriends that he looks pale. Physical examination confirmspale conjunctivae without icterus. His blood pressure is145/86 mm Hg, and his pulse is 92/min. His hemoglobinlevel is 8 g/dL. Further laboratory studies indicate that hehas microcytic anemia, with very low levels of serum ferritin. Which of the following is the most appropriatenext diagnostic step?

(A) Coombs test

(B) Hemoglobin electrophoresis

(C) Occult blood in stool

(D) Schilling test

(E) Serum creatinine

67. A 42-year-old man has been fired from his job becauseof inappropriate behavior. His employer insists that hehas shown up for work drunk, an accusation that thepatient vehemently denies. His case has not been helpedby the fact that he has been arrested twice for drivingunder the influence of alcohol. In both cases the chargeswere dismissed because breath tests did not confirmillegal levels of alcohol. The patient admits that some-times he feels as though he is drunk, he staggers whenhe walks, and his mind “is not clear.” On those occa-sions he also feels intense hunger, and eating foodsnacks seems to eliminate the symptoms. In response todirect questioning, he relates that these episodes neverhappen after he eats, but seem to be triggered by fasting.Except for mild obesity, his physical examination iscompletely normal. Which of the following would bemost helpful to elucidate the diagnosis?

(A) Fasting blood sugar, insulin, and C peptide levels

(B) Funduscopic examination

(C) Liver function tests

(D) MRI of the brain

(E) Psychiatric consultation

68. A 43-year-old woman has an episode of hematemesisand is brought in for evaluation. She is hemodynami-cally stable and her hemoglobin is 13 g/dL. Three yearsearlier she had been diagnosed elsewhere with pepticulcer disease, and she has been to many physicians try-ing to regain her health. She has undergone two inten-sive trials of medical management that have beenunsuccessful. On both occasions she took a full courseof antibiotics to eradicate Helicobacter pylori and wastreated with proton pump inhibitors. Discouraged bysuch failures, she became resigned to her fate and reliedon over-the-counter antacids to alleviate her persistentepigastric pain. More recently she sought help forwatery diarrhea, which initial studies characterized assecretory in nature; but the workup was never complet-ed because she lost her medical insurance. At this time,an upper gastrointestinal endoscopy shows a 2-cm ulceron the posterior wall of the first portion of the duode-num, with a fresh blood clot on it, but not activelybleeding. Beyond that ulcer, there is a second ulcer inthe first portion and two more ulcers in the second por-tion of the duodenal loop. Which of the following is themost appropriate next diagnostic study?

(A) Biopsy of the small bowel mucosa

(B) Biopsy of the ulcer that bled

(C) Fasting serum gastrin levels

(D) Serum calcium levels

(E) Upper gastrointestinal series with barium

Internal Medicine

Page 28: Internal Medicine With Answers

69. A 62-year-old man has massive hematochezia. He had alarge bloody bowel movement 3 hours ago and twomore bloody evacuations since then. On arrival at theemergency department he has a blood pressure of 78/60mm Hg and a pulse of 110/min. He is given 2 L ofRinger lactate in 20 minutes, which brings his bloodpressure to 100/85 mm Hg and his pulse to 105/min.This is followed by type-specific packed red cells assoon as they become available. In the first hour, herequires a total of four red cell packs to maintain stablevital signs. During that time, a nasogastric tube isinserted, which produces a clear, green fluid withoutblood. Anoscopy and proctosigmoidoscopy are alsodone, and except for a brisk flow of bright red bloodcoming from above the instruments, no pathology isdetected and no bleeding lesion is identified. Which ofthe following is the most likely cause of his bleeding?

(A) Colon cancer

(B) Diverticulosis

(C) Duodenal ulcer

(D) Inflammatory bowel disease

(E) Portal hypertension

70. An 81-year-old woman is brought to the emergencydepartment with left lower quadrant pain and fever for48 hours. She was discharged from the hospital 3 daysago after an admission for Escherichia coli urosepsis, forwhich she was successfully treated with gentamicin. Hertemperature is 38.4 C (101.1 F), blood pressure is162/102 mm Hg, and pulse is 88/min. Examinationreveals abdominal tenderness and fullness in the leftlower quadrant with guarding. An abdominal andpelvic CT scan shows an inflammatory mass surround-ing the sigmoid colon in an area of many diverticula.Her leukocyte count is 18,400/mm3, hemoglobin is 13g/dL, hematocrit is 40%, and a platelet count is304,000/mm3. Her sodium is 132 mEq/L, potassium is4.7 mEq/L, bicarbonate is 20 mEq/L, chloride is 110mEq/L, BUN is 33 mg/dL, creatinine is 3.7 mg/dL, andglucose is 114 mg/dL. Which of the following is themost likely explanation for her renal insufficiency?

(A) Acute tubular necrosis

(B) Diverticulitis

(C) Left ureteral obstruction

(D) Pyelonephritis

(E) Septic shock

71. A previously healthy 24-year-old woman has recurrentheadaches. The symptoms began approximately 4 weeks ago, have been present almost daily, and areunrelated to time of day or meals. Her temperature is 37C (98.6 F), blood pressure is 156/96 mm Hg, pulse is78/min, and respirations are 14/min. Funduscopicexamination is normal. There are no carotid bruits. Herlungs are clear. She has a regular heart rhythm withoutmurmurs or extra heart sounds. On abdominal exami-nation, a left upper quadrant bruit could be heard. Shehas no hepatosplenomegaly. Her pulses are full andsymmetric in all regions. On two separate evaluationsover the next week, her blood pressure measuresbetween 155 and 170 mm Hg systolic and 95 and 105mm Hg diastolic. There is no blood pressure gradientbetween her arms and legs. Which of the following isthe most likely explanation for her hypertension?

(A) Aortic dissection

(B) Coarctation of the aorta

(C) Hypertrophic cardiomyopathy

(D) Renovascular hypertension

(E) Takayasu arteritis

72. A 20-year-old college student develops a severeheadache, nausea, photophobia, and myalgias. His tem-perature is 39.5 C (103.1 F), pulse is 105/min, bloodpressure is 110/65 mm Hg, and respirations are 26/min.He appears acutely ill and is drowsy. Funduscopicexamination is normal. There are no focal motor orsensory deficits. A lumbar puncture yields cloudy cere-brospinal fluid. A Gram stain of the fluid would mostlikely reveal leukocytes and which of the followingorganisms?

(A) Encapsulated yeast with hyphae

(B) Gram-negative bacilli in pairs

(C) Gram-negative cocci in clusters

(D) Gram-positive bacilli in pairs

(E) Gram-positive cocci in pairs

26

USMLE Step 2 Assessment Exam

Page 29: Internal Medicine With Answers

27

73. A 62-year-old woman with a history of stable angina,hyperlipidemia, and a 35-pack-year tobacco historycomes to the emergency department with substernalchest pain. The pain began 45 minutes ago at rest and hasnot been relieved by three sublingual nitroglycerintablets. The pain is nonradiating and is associated withshortness of breath, nausea, and vomiting. Her bloodpressure is 160/100 mm Hg and pulse is 100/min.Cardiovascular examination shows a left-sided S3. Anelectrocardiogram shows sinus tachycardia with 5-mmST segment elevations in leads V1 to V6. While aggressivelowering of blood pressure and pulse is begun, which ofthe following is the most appropriate course of therapy?

(A) Admission to the coronary care unit for exclusionof myocardial infarction

(B) Immediate referral for emergent coronary arterybypass surgery

(C) Immediate reperfusion with thrombolysis

(D) Immediate synchronized cardioversion

(E) PET scan of the myocardium to determine extentof ischemia

74. A healthy 30-year-old man with an unremarkable his-tory and physical examination has a PPD placed beforestarting medical school. Two days later, the skin at thatsite shows 8 mm of induration. He has never had a PPDplaced before and has never been diagnosed or treatedfor tuberculosis. In addition, he has no risk factors fortuberculosis, was born and has lived his entire life in theUnited States, and has never received the bacillusCalmette-Guerin vaccine. Which of the following is themost appropriate next step in management?

(A) No further management is needed

(B) Prescribe isoniazid, rifampin, and pyrazinamideisoniazid for 6 to 12 months

(C) Prescribe isoniazid for 6 to 12 months

(D) Prescribe rifampin for 6 to 12 months

(E) Refer him for bronchoscopy to confirm the result

75. A 32-year-old woman comes to the clinic one cold dayin January complaining of fever, chills, headache, myal-gias, and malaise. She also feels she is developing acough and sore throat. Her symptoms seemed to havebegun last night but were more severe this morning.She denies any major medical history and has never hadany respiratory problems such as asthma or pneumo-nia. She has never been vaccinated for influenza. It isnotable that many of her coworkers have been ill andhave had similar symptoms. Physical exam reveals somemild oropharyngeal hyperemia, mild cervical adenopa-thy, and clear lung fields, though the patient has acough on end expiration. The patient asks if there is anyavailable treatment because she is extremely busy and“every day counts.” In addition to supportive care,which of the following is a treatment that will shortenthe duration of this patient’s illness?

(A) Amantadine if influenza titers are elevated

(B) Empiric macrolide antibiotics

(C) Empiric inhaled oseltamivir

(D) High-dose vitamin C and zinc lozenges

(E) No additional treatment is available

Internal Medicine

Page 30: Internal Medicine With Answers
Page 31: Internal Medicine With Answers

1. C

2. D

3. C

4. E

5. E

6. C

7. E

8. B

9. E

10. D

11. E

12. A

13. B

14. E

15. B

16. B

17. E

18. C

19. A

20. D

21. C

22. B

23. B

24. B

25. C

26. D

27. A

28. A

29. A

30. A

31. C

32. C

33. C

34. B

35. C

36. C

37. C

38. D

39. E

40. E

41. B

42. A

43. E

44. B

45. C

46. C

47. E

48. A

49. E

50. D

51. B

52. C

53. C

54. C

55. D

56. E

57. D

58. D

59. B

60. B

61. A

62. D

63. C

64. C

65. D

66. C

67. A

68. C

69. B

70. A

71. D

72. E

73. C

74. A

75. C

Internal Medicine Assessment Exam Answers and Explanations

29

ANSWER KEY

Page 32: Internal Medicine With Answers

1. The correct answer is C. This patient has hypertensionand hypokalemia, two of the cardinal symptoms of pri-mary hyperaldosteronism (Conn syndrome). He alsohas slightly high serum bicarbonate, suggestive of ametabolic alkalemia, which is also consistent with thisdiagnosis. Routine serum electrolytes and BUN/creatinine are part of the workup of hypertension; thisis done primarily to rule out the presence of hyperal-dosteronism, Cushing syndrome, and intrinsic renaldisease. Once the disease is suspected on clinicalgrounds, the test of choice to screen for Conn syn-drome is a plasma renin activity to aldosterone ratio. Alevel greater than 30 in the appropriate clinical contextis highly suggestive of the disease and warrants defini-tive testing that involves a volume challenge with nor-mal saline followed by a repeat assay for aldosterone.Patients with confirmed hyperaldosteronism shouldhave a full-body contrast CT scan to search for thetumor and should have it resected, if possible.

An abdominal CT scan with contrast (choice A) wouldbe helpful in looking for an abdominal mass, especiallyan adrenal one. Before such a search is undertaken,however, the presence of a metabolically active tumormust be confirmed. Adrenal and other incidentalomasare common and often are not the cause of the condi-tion being investigated.

A low-dose dexamethasone suppression test (choice B)is a common initial test for Cushing syndrome. In thistest, the patient takes one 1-mg dose of dexamethasoneat midnight and has a serum cortisol level drawn at 8 A.M. the following day. An A.M. cortisol of less than 5 µg/dL essentially excludes Cushing syndrome and is agood initial screening test for the disorder. In thispatient, the negative review of systems and absence ofphysical findings other than hypertension makeCushing syndrome less likely than Conn syndrome.

Renal biopsy (choice D) is a test best reserved forpatients with elevated creatinine and abnormal urinal-ysis as part of the workup for intrinsic renal disease.

A 24-hour urine cortisol (choice E) is the screening testof choice for patients suspected of having Cushing syn-drome. The sensitivity of this test is poor, though, as isthe specificity. Because excess cortisol exerts its hyper-tensive and potassium-wasting effect primarily throughits action on the mineralocorticoid receptor (just likeexcess aldosterone), the syndrome can present similarlyto hyperaldosteronism. Other physical findings, however,such as the buffalo hump, moon facies, purple abdom-inal striae, proximal muscle wasting, abdominal obesity,and mood changes are often present and should beactively sought during the routine history and physicalexamination for hypertension.

2. The correct answer is D. This patient has COPD, orchronic obstructive pulmonary disease. The diseaseoccurs in up to 15% of long-term (>20 pack-years)smokers and is a clinical diagnosis, though there aredistinct histopathologic correlates. The diagnosisrequires a chronic, productive cough of at least 3months’ duration over 2 consecutive years. Thesepatients have airtrapping that worsens as the diseaseprogresses. The result is hyperinflated lungs. This man-ifests as an increase in TLC (total lung capacity) andFRC (functional residual capacity), not a decrease inFRC (choice A) and RV (residual volume), thoughthere is often a decrease in VC (vital capacity).

The ratio of FEV1 to FVC provides one means to dis-tinguish between restrictive disease, such as interstitiallung disease, and obstructive conditions, such asCOPD. In obstructive disease, the FEV1 decreases to asignificantly greater degree than the FVC, resulting in areduced FEV1/FVC ratio. In restrictive disease, there isan equivalent decrease in FVC (choice B), resulting in anormal ratio. This patient, with evidence of airwayobstruction, such as prolonged expiratory phase of res-piration, pursed lip breathing, and expiratory wheeze,would likely have a much lower FEV1 and FVC.

The DLCO often decreases rather than increases(choice C) in COPD. Emphysematous changes result inalveolar surface loss, which reduces the area availablefor carbon monoxide diffusion. Conditions that occa-sionally increase DLCO include acute asthma and earlycongestive heart failure.

Flow-volume loops, which are tracings representing apatient’s inspiratory and expiratory effort, can showevidence of restrictive or obstructive ventilatorydefects. Restrictive, not obstructive, defects create ashort and narrow tracing (choice E). The reduction ofair and lung volumes results in a quick and short expi-ration. In contrast, obstructive defects are often wideand caved, representing a prolonged expiratory phase.

3. The correct answer is C. This patient has evidence ofan acute viral bronchitis. This condition, whichaccounts for 10 million office visits a year, is clinicallydefined as an acute respiratory illness lasting less than 3weeks and characterized primarily by cough. The vastmajority of cases are due to viruses and, in healthyhosts, do not require further workup. The acute onsetof symptoms (less than a week) and sick-contact with achild who likely has a viral infection both argue in favorof a viral etiology. Symptomatic treatment includesantitussive agents such as dextromethorphan, decon-gestants, antipyretics, analgesics, and, if bronchialhyper-responsiveness is present (if the patient has awheeze), inhaled beta-agonists.

30

USMLE Step 2 Assessment Exam

Page 33: Internal Medicine With Answers

31

It is not necessary to routinely order chest radiographs(choice A) for patients with a cough. If pneumonia issuspected, or if the patient has other risk factors(known lung disease, long tobacco history, immuno-suppression, etc.), a radiograph is appropriate. In gen-eral, a chest radiograph should be ordered if any of thefollowing criteria are met: pulse >100/min, respiratoryrate >24/min, oral temperature greater than 38 C(100.4 F), an abnormal chest exam, or if there is an epi-demiologic suspicion of SARS.

Empiric macrolide antibiotics (choice B) are frequentlyused for patients with COPD who have a worsening ofbronchitis. These patients have structural lung diseaseand altered ciliary functioning that increase their likeli-hood of having bacterial pathogens exacerbating theirrespiratory problems.

Sputum culture and Gram stain (choice D) are notappropriate tests, given the high pretest probability of aviral etiology.

Viral serologies and culture (choice E) are not appro-priate because the results are usually not available untilafter the illness has resolved and are unlikely to changemanagement of this apparent case of uncomplicatedacute bronchitis.

4. The correct answer is E. This patient suffers fromMénière disease, an idiopathic, episodic cause of vertigoand hearing loss. Multiple medications have provenineffective in treating this, and it is usually very difficultto distinguish the natural waxing and waning of diseaseseverity from a treatment response.

Complete loss of vestibular sensation of balance (choiceA) is a very rare disorder that requires extensive damageto the vestibular apparatus (the utricle, saccule, andsemicircular canals). This is seen most commonly inoto/vestibulotoxicity secondary to aminoglycosideantibiotics or to loop diuretics.

Experiencing complete resolution of symptoms (choiceB) occurs occasionally in Ménière disease, especially ifit is associated with an acute viral infection (labyrinthi-tis). If they truly had a case of chronic Ménière disease,though, only a minority has complete resolution.

Experiencing a continued increase in the frequency andseverity of vertigo symptoms (choice C) occurs onlyrarely in patients with Ménière disease. Observationalstudies have found that this occurs in only approxi-mately 10% of patients.

Development of refractory tinnitus (choice D) is anuncommon complication of Ménière disease. As withother manifestations, it stems from permanent damageto the cochlear hair cells. Over the long term it can even

induce neuropathic changes in the vestibulocochlearnerve and more central auditory nuclei. This can makethe tinnitus refractory even to ablation of the vestibulo-cochlear nerve (central tinnitus).

5. The correct answer is E. This patient has manifesta-tions of systemic lupus erythematosus (SLE). Althoughanti-Sm antibody is positive in only approximately onequarter to one third of cases of SLE, it is highly specificfor the condition; no other diagnosis routinely resultsin the presence of this antibody. This is in contrast toantinuclear antibody (choice C), which is highly sensi-tive but not very specific, being present in a variety ofconditions. The presence of this anti-Sm is more com-mon in African American females with SLE and haspoor prognostic significance. It is often associated withCNS symptoms and lupus nephritis.

Anticentromere antibodies (choice A) are associatedwith CREST syndrome, which manifests as calci-nosis, Raynaud phenomenon, esophageal dysmotilitysymptoms, and telangiectasias. The test is not highlysensitive for the condition but is fairly specific forCREST syndrome.

Antihistone antibodies (choice B) are sensitive indica-tors of drug-induced lupus, as they are present in mostcases of the disease. Antihistone antibodies are also pre-sent in SLE, however, though they are neither a sensitivenor specific marker of the disease.

Anti-Scl 70 (choice D), or antitopoisomerase-1 anti-bodies are highly specific, though not very sensitive, forsystemic sclerosis. They are not specific for SLE.

6. The correct answer is C. This patient has primaryRaynaud phenomenon, that is, Raynaud phenomenonin the absence of any other rheumatologic disease. Thesymptoms are typical and the disease most commonlyaffects women between the ages of 20 and 50 years. Thesyndrome is caused by vasospasm of local arterioles,leading to pallor followed by cyanosis and pain.Eventually the vasospasm ceases and a reactive hyper-emia occurs. Dihydropyridine calcium channel block-ers are the most effective means of decreasing thesymptoms of Raynaud phenomenon and resulted insymptomatic improvement over 4 to 12 weeks in sixrandomized controlled trials. In a prospective cohort,only 13% of patients who initially presented with iso-lated Raynaud phenomenon went on to develop arheumatologic disorder such as lupus or scleroderma.The syndrome often remits on its own but should betreated symptomatically while appropriate workup fora rheumatologic cause proceeds.

Internal Medicine Answers and Explanations

Page 34: Internal Medicine With Answers

Amitriptyline (choice A) is a tricyclic antidepressantthat at low doses is effective in treating neuropathy andneuropathic pain in a variety of diseases. It also has theside effect of orthostatic hypotension because of itsaction on alpha-1 adrenergic receptors in vascularsmooth muscle. Its vasoactive properties, however, arenot sufficient to be of use in Raynaud phenomenon,and neuropathy is not the cause of this patient’s pain.

Gabapentin (choice B) is used widely for a variety ofneurologic complaints, almost none of which have beenstudied in rigorous, controlled, randomized trials. Itseffectiveness in vascular phenomena such as Raynaudphenomenon is unknown.

Prazosin (choice D) is also a potent antagonist of thealpha-1 adrenergic receptor in vascular smooth muscleand is effective in the treatment of hypertension for thisreason. It has been studied in the treatment of primaryRaynaud phenomenon and was found to be ineffectivein one small crossover randomized trial. Attacks wereless frequent but were no less severe when comparedwith placebo.

Verapamil (choice E) is a nondihydropyridine calciumchannel blocker that has its primary effect on cardiacmuscle and the cardiac SA and AV nodes. It has very littleeffect on peripheral vascular smooth muscle and has notbeen studied in the treatment of Raynaud phenomenon.

7. The correct answer is E. This patient likely hasHuntington chorea. Although the diagnosis is often diffi-cult to make, the combination of chorea, neuropsychiatricsymptoms, age of onset, and a family history of diseaseoccurring in middle adulthood solidify the diagnosis.Genetic testing, which will reveal the evidence of trinu-cleotide repeats that produce a neurotoxic huntingtinprotein, is available to confirm such a diagnosis.

Abnormal copper regulation (choice A) is the cause ofWilson disease, not Huntington chorea. Wilson diseasemay also present with early behavioral changes, but ismore likely to occur at a young age, does not result inmovement disorders, and is associated with liver dys-function.

Dopaminergic loss in the substantia nigra (choice B),along with the presence of Lewy bodies, is associatedwith Parkinson disease. Parkinson disease manifestspredominantly with “brady” symptoms, such as slowedgait, slowed thought, and a slow tremor. Chorea is theopposite of these bradykinetic symptoms, and is due toa relative excess of dopamine in the basal ganglia.Although patients with Huntington may have parkin-sonian symptoms, the underlying cause is from trinu-cleotide repeats that result in primarily frontostriatalnerve destruction.

Elevated total body iron stores (choice C) are the causeof hemachromatosis. This disease will often cause heartfailure, liver disease, and erectile dysfunction, but itdoes not commonly manifest as overt neuropsychiatricsymptoms.

Extracellular deposition of beta-amyloid and neurofib-rillary tangles (choice D) is consistent with Alzheimerdementia, an unlikely diagnosis in this patient.Alzheimer tends to occur in older patients and presentswith early memory impairment rather than movementdisorder.

8. The correct answer is B. Patterns of liver function teststend to correlate with the pathophysiology of differentdisease states. Alcoholic hepatitis classically results in atransaminitis that is manifested as an AST at least twicethe ALT. The ALT is usually less than 300 U/L andalmost always less than 500 U/L. The important caveatis that if the patient already has severe cirrhosis or end-stage liver disease, these ratios may not hold true. Incontrast, viral hepatitis often has an AST level less thanthe ALT level, with both being greater than 1000 U/L(choice A). If the aminotransferase levels are severelyelevated, however (in the 2000-3000 U/L range orgreater) (choice C), another diagnosis should be con-sidered. Shock liver and acetaminophen overdose aretwo common explanations for extremely high amino-transferase levels.

In some cases of hepatitis, one can find near normalalkaline phosphatase and gamma-glutamyltransferaselevels (choice D). Severe hepatitis and particularly alco-holic hepatitis, however, tends to result in elevations ofboth values.

Often included in liver function tests are the total pro-tein level and albumin level. The total protein is com-posed of albumin together with circulatingimmunoglobulin and various other carrier proteins. Inliver disease, the albumin and thus the total protein areoften low. If the total protein remains elevated and ismore than 2 to 3 times the albumin level (choice E),another source of “excess” protein needs to be found.Often, disease states that cause chronic antigenemia orincreased immunoglobulin production result in anincreased total protein to albumin ratio. HIV infection,hepatitis B virus, and multiple myeloma are commoncauses of this abnormality.

9. The correct answer is E. This woman has isolatedhypercalcemia and a symptomatic urinary tract stone.Symptoms of hypercalcemia include those attributableto renal stones, as this patient currently has.Hypercalcemia also causes psychiatric problems such as

32

USMLE Step 2 Assessment Exam

Page 35: Internal Medicine With Answers

33

mood instability, anxiety, and cognitive problems,together with diffuse bone pain, constipation, andvague periumbilical abdominal pain. The most likelycause of hypercalcemia in young, otherwise healthypeople is primary hyperparathyroidism, usually sec-ondary to a parathyroid adenoma.

Chronic diarrhea (choice A) would not be an expectedsymptom. High levels of serum calcium causedecreased smooth muscle tone and autonomic dysfunc-tion in the gut, causing constipation rather than diar-rhea.

Exertional dyspnea (choice B) has not been described asa symptom of hypercalcemia. Patients often have mildgeneralized weakness that improves after correction ofthe cause of hypercalcemia.

Swollen, painful joints in the fingers and toes (choice C)are symptoms of osteoarthritis, which has not been con-nected with hypercalcemia. Primary hyperparathy-roidism often causes diffuse bone pain because of rapidbone turnover and can rarely lead to cases of pseudo-gout because of precipitation of calcium pyrophosphatecrystals in joints. Joint manifestations are rare, however,and pseudogout typically involves the large joints suchas the knees and hips rather than the fingers and toes.

Muscle tetany (choice D) is a finding in hypocalcemia,not hypercalcemia. This finding is known asTrousseau’s sign and results from the combination ofhypocalcemia and ischemia caused by inflating theblood pressure cuff. Another finding in hypocalcemia isChvostek’s sign, in which percussion over the facialnerve causes twitching of the corresponding muscles ofthe face. Physical examination of patients with knownhypercalcemia is usually completely unrevealing. Evenpalpation of the thyroid bed for adenomas seldomreveals the causative parathyroid adenoma.

10. The correct answer is D. Glucose creates a powerfulosmotic force that can pull water out of cells and intothe intravascular space, resulting in a dilutionalhyponatremia. In cases of severe hypoglycemia, thisdilutional effect can result in significant hyponatremia,at least by laboratory testing. The sodium level needs tobe adjusted for the elevated blood glucose. Basically, forevery 100 mg/dL above 100, the sodium level decreasesby 1.6 mEq/L. The adjusted serum sodium, then, is:

Measured sodium + (glucose – 100)/100 × 1.6 = corrected serum sodium

Albumin (choice A) needs to be measured to determinethe true serum calcium. Because calcium is tightly pro-tein bound, the total level of calcium increases anddecreases with albumin levels, whereas the ionized,active portion stays constant.

Alcohol (choice B) can cause a ketoacidosis but usuallycauses a hypoglycemia. It is metabolized rapidly by cellsand generally is not an effective osmole. It is unlikely toexplain this patient’s sodium level.

The creatinine level (choice C) is important to know, asinsulin is cleared renally. For nondialysis patients, how-ever, this value does not assist in understanding thepatient’s hyponatremia.

Potassium (choice E) needs to be followed closely whentreating hyperglycemia because patients can becomehypokalemic quickly. This value does not affect theserum sodium level directly, however.

11. The correct answer is E. He passed a uric acid stoneand the remaining stone that he has not passed yet hasthe radiologic characteristics of another uric acid stone.Uric acid stones can be dissolved medically by alkalin-ization of the urine.

Acidification of the urine (choice A) is the opposite ofwhat will dissolve the stone.

Endoscopic extraction (choice B) or shock wavelithotripsy (choice D) does not need to be consideredyet, because more conservative medical managementcan be expected to solve the problem.

Antibiotic therapy (choice C) has a role to play in themanagement of struvite stones in which chronic infec-tion of the urinary tract is an etiologic factor.

12. The correct answer is A. This patient likely has primarysclerosing cholangitis (PSC), an autoimmune-mediatedinflammation and destruction of the large bile ductsthat results in fibrosis and stricture. It is associated com-monly with inflammatory bowel disease, and althoughpartially treatable with ursodeoxycholate, is relentlesslyprogressive. Definitive treatment is liver transplantation.

All that is biliary is not gallstones. Recurrent gallstones,either cholesterol (choice B) or pigment (choice E), area less likely cause of this patient’s symptoms than is PSC.Although he has no major risk factors for gallstones, hedoes have a major risk factor for PSC, together with hav-ing a serum marker of autoimmune disease, a positiveANCA. Often an antinuclear antibody assay and an anti-smooth muscle antibody test also are positive.

This patient is at increased risk for colon cancer (choiceC), though Crohn disease is less associated with earlycolon cancer than it is with ulcerative colitis. Thispatient, given an unremarkable review of symptoms, ismore likely to have PSC than a large, metastatic coloncancer causing biliary obstruction.

There is no reason to suspect a hepatoma (choice D),which is unlikely to present primarily as a cholestaticpicture.

Internal Medicine Answers and Explanations

Page 36: Internal Medicine With Answers

13. The correct answer is B. This patient has an increase inheartburn symptoms and abdominal pain. Dyspepsia isa common complaint, and the most common causes aregastroesophageal ulcers/erosions, atypical gastroe-sophageal reflux disease, nonulcer dyspepsia, and gas-tric/duodenal cancer. The diagnostic concern here isgastric cancer, the incidence of which increases withage. In patients older than age 45 years, endoscopy isrecommended as the initial screening test of choice. Ithas the advantages of excellent sensitivity, the ability toperform biopsies for ulcers and Helicobacter pylori test-ing, and potential therapeutic intervention if an ulcer isfound to be bleeding. If an ulcer or other benignpathology is defined, the patient can be given antibi-otics for H. pylori eradication or can be given a proton-pump inhibitor for his symptoms.

Empiric therapy with a proton pump inhibitor (PPI)(choice A) is only acceptable for younger patientswhose risk for gastric cancer is very low. In thesepatients, response to a PPI is adequate to make the diag-nosis of dyspepsia, and this therapy is likely to causehealing of any ulcers that are present. Testing for H.pylori through serology or stool antigen may be of ben-efit, as eradication of the organism with antibioticsleads to a better chance of resolving the symptoms.Should a younger patient fail to respond completely tothis therapy or if they have warning symptoms such asbleeding, fatigue, weight loss, or dysphagia, they shouldbe referred for early endoscopy.

Fecal occult blood testing (choice C) helps assesswhether the patient is losing blood from a gastroduo-denal ulcer or a gastric cancer. If the patient is notactively bleeding from such a lesion, however, this test isfalsely negative. Again, in older patients with new-onsetdyspepsia, the concern for gastric cancer merits aninvasive workup with EGD.

Serology for H. pylori (choice D) is an appropriate testin younger patients and helps to determine whethereradication therapy is needed. It provides no informa-tion, however, about other potential causes of thispatient’s pain.

An upper gastrointestinal series (choice E) is incorrectbecause of its low sensitivity for gastric cancer, especiallyfor early lesions that may be amenable to therapy. Its sen-sitivity for ulcers is also fairly low and does not allowbiopsies to be performed if a lesion is found.

14. The correct answer is E. This patient has chronic mye-logenous leukemia, definitively diagnosed by the pres-ence of the Philadelphia chromosome, t(9;22). Medicaltreatment options are varied, and the exact protocolsare complex and evolving. However, young patients

with available donors and no major comorbidities dobetter with stem cell transplant than with purechemotherapy. As such, the definitive treatment for thispatient is hematopoietic cell transplantation from anHLA-matched donor.

Cytarabine and busulfan (choice A), as well as hydrox-yurea (choice B), are chemotherapeutic agents that canbe combined with interferon-alpha (choice C).Interferon-alpha–based regimens are highly effectivetreatments that can induce remission. Recently, oraltyrosine kinase inhibitors have been found to be supe-rior to some of these traditional treatments. However,in young patients, stem cell transplantation is still thedefinitive treatment. Interferon-beta (choice D) is notused to treat CML.

15. The correct answer is B. An isolated GGT elevationmay be the only laboratory abnormality seen in a clini-cally stable alcoholic patient. Although the test is by nomeans sensitive enough to use for screening purposes,an isolated GGT level (relative to alkaline phosphataseand other liver function tests) should raise the possibil-ity of alcohol abuse. This patient was recently firedfrom work (raising the question why), reports depres-sive symptoms, and has rhinophyma, a physical findingloosely associated with alcohol use. Other drugs thatcan cause an isolated GGT elevation include barbitu-rates and phenytoin.

Acetaminophen (choice A) toxicity will present withmassive transaminitis, with levels in the thousands. Anisolated GGT elevation, which can indicate early liverdisease, is not consistent with acetaminophen toxicity.

In many patients, amiodarone (choice C) can cause amoderate increase in serum transaminases and, rarely,more severe hepatic injury. Hepatomegaly is usuallypresent, and a transaminitis, not an elevation ofcholestatic markers such as GGT, is more likely.

Atorvastatin (choice D), like all statins, can cause ele-vations of liver enzymes, though they often return tonormal. Again, the pattern is a transaminitis, not anisolated elevation of GGT.

Metformin (choice E), which acts primarily by limitinghepatic gluconeogenesis, needs to be used cautiously inpatients with liver failure. It is not, however, commonlyassociated with hepatic abnormalities.

16. The correct answer is B. This patient is not stable.Altered mental status, hypotension, and a weak, threadypulse warrant immediate action. The first interventionin ventricular tachycardia is early defibrillation. Thepatient should be shocked with 3 shocks. After thesestacked shocks, other medications such as amiodarone

34

USMLE Step 2 Assessment Exam

Page 37: Internal Medicine With Answers

35

(choice A), epinephrine (choice C), or lidocaine (choiceD) can be used. The mnemonic for remembering thesequence of events in treating unstable v-tach is,“Shock, shock, shock, everybody shock, and let’s makepatients better.” This stands for three stacked shocks,epinephrine, shock, then amiodarone, lidocaine, mag-nesium, and procainamide.

This patient is not in a stable rhythm, and should betreated now. Given that he is already on a cardiac mon-itor, and assuming a crash cart is available, not muchwill be gained by an immediate transfer to a cardiaccare unit (choice E).

17. The correct answer is E. This patient likely has perni-cious anemia, a disease characterized by autoimmunedestruction of the parietal cells that secrete intrinsicfactor, which is necessary for vitamin B12 absorption.The “seven clinical Ps” of pernicious anemia are pan-cytopenia, peripheral neuropathy, posterior spinal col-umn neuropathy, pyramidal tract signs, papillary(tongue) atrophy, pH elevation (gastric fluid), and psy-chosis (megaloblastic madness). The classic diagnostictest to confirm the clinical suspicion of perniciousanemia is a Schilling test. Basically, radiolabeled cobal-amin is administered orally after a parenteral dose hassaturated all the cobalamin binding sites. The urine isthen checked for excreted radiolabeled cobalamin. Ifthere is cobalamin in the urine, then absorption is ade-quate and the patient is unlikely to have perniciousanemia. If there is no absorption (i.e., no urinarycobalamin), another dose of cobalamin is given, thistime with coadministration of intrinsic factor. If thereis absorption this time, then the diagnosis of perni-cious anemia (which is due to a deficiency of intrinsicfactor) is almost certain. If there is no absorption,small gut pathology (Crohn, ileal resection, bacterialovergrowth, etc.) should be suspected.

A bone marrow biopsy (choice A) will show erythroidhyperplasia, megaloblastic cell lines, and normal orhigh iron, but is not necessary for the diagnosis of thisdisease. Although it can point toward folate and B12deficiencies (but cannot differentiate the two), it willnot diagnose the cause of the deficiency.

The Ham test (choice B) is an acidification test used totest for paroxysmal nocturnal hemoglobinuria, a rarecause of hemolytic anemia. It is not an appropriate test forthis patient, who does not appear to have a hemolyticanemia.

Lumbar puncture (choice C) and MRI (choice D) canbe useful if this patient’s primary problem is believed tobe a CNS lesion, such as demyelination from multiplesclerosis. However, the peripheral blood smear should

point toward a hematologic cause of this patient’s prob-lems. As such, these are not the most appropriate firsttests to order.

18. The correct answer is C. This patent has a chylous effu-sion, defined as a lymphatic leak into the pleural space.There are a variety of causes of chylous effusions, fallingmainly into the categories of traumatic and nontraumat-ic. Traumatic causes include surgery, in which the tho-racic duct can be nicked (this patient has no surgical his-tory) and thoracic trauma. Nonsurgical causes includeinfection and, much more commonly, malignancy. Themost common malignancy to cause a chylothorax is lym-phoma, as hinted at by this patient’s B-symptoms andabnormal chest radiograph.

There is no reason to suspect an acid-fast organism(choice A) such as tuberculosis (TB). Such infectionsrarely cause a chylous effusion and, although some ofthis patient’s symptoms may be attributed to TB, he hasno cough or evidence of TB on chest radiograph.

Lymphangioleiomyomatosis (choice B) is an extremelyrare disease that affects women of childbearing age andmay cause chylous effusions. It is an unlikely diagnosisin this middle-aged man.

Pancreatic disease, including adenocarcinoma (choiceD), can cause pleural effusions and ascites, though itdoes not cause chylous effusions.

This patient has no risk factors for mesothelioma(choice E), which is associated almost exclusively withasbestos exposure. Further, such a condition, although itmay cause an exudative malignant effusion, is unlikelyto cause a chylous effusion.

19. The correct answer is A. Although a radioactive uptakescan is not necessary to diagnosis this patient’s condi-tion, subacute thyroiditis, it is a common way to thinkof the pathogenesis of hyperthyroidism. Conditionsthat increase thyroid activity, such as Graves disease,can be expected to result in increased incorporation ofiodine and will result in increased uptake of theradioactive tracer. Conditions that result in increasedthyroid hormone without increasing the production ofhormone, basically exogenous hormone (sometimestaken for weight loss), or inflammatory damage of thethyroid and release of colloid, result in reduced uptake.The excess thyroid hormone results in feedback inhibi-tion of TSH, resulting in less uptake of iodine. Subacutethyroiditis, in which thyroxine is “spilled” from aninflamed thyroid gland, can result in frank hyperthy-roidism with a reduced uptake of radioactive iodine.

A focal area of increased uptake with the rest of thegland revealing reduced uptake (choice B) is consistent

Internal Medicine Answers and Explanations

Page 38: Internal Medicine With Answers

with a toxic adenoma. The adenomatous tissue secretesthyroxine free from TSH-mediated feedback. The thy-roxine reduces TSH secretion, however, which doesaffect the normal portion of the gland. Without TSH,the rest of the gland does not incorporate iodine readi-ly. Multinodular toxic goiter presents similarly, but withmultiple nodular areas of increased uptake surroundedby areas of reduced uptake (choice D).

Increased uptake throughout the gland (choice C) indi-cates an increased amount of colloid formation, aswould be seen in conditions such as Graves disease, notsubacute thyroiditis.

Given this patient’s symptoms and abnormal thyroidfunction tests, a normal scan (choice E) is extremelyunlikely. Given a low TSH level, at least part of thegland will act abnormally, as the stimulus to incorpo-rate iodine is not present.

20. The correct answer is D. In all states, hepatitis A is areportable illness. Given the ease of transmission, andthat most patients can transmit the disease to familymembers, public health officials need to be notifiedsooner rather than later.

This patient should have immunity to hepatitis A, anddoes not appear to have risk factors for recurrent infec-tion in any case. In most patients, IgM levels aredetectable 5 to 10 days before symptoms, with IgG lev-els rising later in the course of the infection. Hepatitis Avaccination (choice A) is not necessary. Similarly, inas-much as this patient already has hepatitis B antibodies,hep B vaccination (choice B) is not necessary.

Liver biopsy (choice C) is inappropriate because hepati-tis A, unlike B and C, does not cause chronic disease.Liver biopsy is used in some risk-stratification algo-rithms for approaching the diagnosis and treatment ofhepatitis C. In addition, some physicians use a rightupper quadrant ultrasound and measure alpha-fetopro-tein levels (choice E). Again, given the natural course ofhepatitis A, this degree of testing is inappropriate.

21. The correct answer is C. This patient has uremic peri-carditis. The “squeaky” noise heard on exam is a classicthree-phase friction rub. Uremic pericarditis, unlikeother forms of pericarditis, is not associated with classicpericarditis EKG findings, such as diffuse ST elevationsand PR depressions. Although the pathophysiology ofuremic pericarditis is unclear, it is related to toxinbuildup from reduced renal function and is thus an indi-cation to start dialysis in a patient who might otherwisehave been watched closely.

Calcium gluconate (choice A) is used for severe hyper-kalemia because it serves to bind to Na/K membrane

pumps, stabilizing the cell membrane. It is not appro-priate in this patient with mild hypokalemia and noclassic EKG changes.

Cardiac catheterization (choice B) and heparin (choiceD) are not appropriate. This is not an acute coronarysyndrome and should not be treated as such. Further, ifthere is a pericardial effusion, heparin can place thepatient at an increased risk for tamponade from hem-orrhage. Note that troponin levels are often elevated inrenal insufficiency.

Indomethacin (choice E) is appropriate for many caus-es of pericarditis but not for uremic pericarditis.Dialysis is the definitive treatment. Further, whateverresidual renal function this patient has can be destroyedby NSAIDs, which reduce the glomerular filtration rate by inhibiting afferent arteriole dilatation, aprostaglandin-mediated event.

22. The correct answer is B. This patient’s pain seems to beesophageal in origin. GERD and esophageal spasm arecommon causes of noncardiac chest pain and oftenhave atypical presentations that mimic coronary chestpain closely. Esophageal spasm often responds to nitro-glycerin, which can relax the esophageal smooth mus-cle. Intermittent dysphagia associated with chest painand a history of GERD strongly suggests esophagealdysfunction. In this setting, a pH probe and outpatientmanometry is an appropriate next diagnostic step.

Cardiac catheterization and angiography (choice A) arenot appropriate. This patient’s only cardiac risk factor ishypertension. Given her young age and completely nor-mal cardiac evaluation, cardiac catheterization isunlikely to be a high yield study and is also highly inva-sive. Similarly, given the normal thallium perfusionscan, further imaging of the heart with a transthoracicechocardiogram (choice C) is unnecessary.

If this patient were not on GERD therapy, an 8-weektrial of a proton pump inhibitor (PPI) would be anappropriate first step in the management of suspectedesophageal chest pain. Given that this patient is alreadyon a PPI, the addition of an H2 blocker (choice D) isunlikely to be beneficial.

Were the dysphagia symptoms constant, endoscopy(choice E) would be an appropriate first step. Routineendoscopy in the evaluation of suspected esophagealchest pain, however, is not recommended. Only 10% ofpatients have esophagitis, and conditions such as GERDor spasm are better evaluated with other modalities.

23. The correct answer is B. Desmopressin is an antidi-uretic hormone (ADH) analogue that can be used totreat central diabetes insipidus (DI), the cause of this

36

USMLE Step 2 Assessment Exam

Page 39: Internal Medicine With Answers

37

patient’s hypernatremia. Hypernatremia in the settingof neurosurgical procedures affecting the pituitary orhypothalamic tract warrants an evaluation for DI. Mildcases of DI often resolve spontaneously, as the hypotha-lamic-pituitary access regains function. Severe damageto the pituitary, however, may result in a triphasicresponse. Initially the patient may have abrupt onset ofDI, which then seems to resolve slowly. This is causedby an initial inhibition of ADH release, followed byinappropriate ADH response from a degenerating pos-terior pituitary. Finally, as stores of ADH are depleted,the patient once again presents with a picture of DI. Assuch, DI needs to be followed closely with serial sodiumlevels. If one is suspicious of the diagnosis, it can beconfirmed with a water restriction test. Water restric-tion in a patient with normal homeostatic mechanismsshould result in an increase in urine osmolality as thebody attempts to retain free water. If the increase isinappropriately minimal and then improves with vaso-pressin (ADH), the diagnosis of central DI is highlylikely.

Demeclocycline (choice A) is a tetracycline-type antibi-otic that blocks ADH, and is used, understandably, fortreating the syndrome of inappropriate antidiuretichormone (SIADH) release. It would likely worsen thispatient’s condition.

Furosemide (choice C) is a loop diuretic. Patients tendto lose more water than salt. Indeed, the combination ofsalt tablets and loop diuretics can be used to treathyponatremia caused by SIADH. As such, it should notbe used to treat hypernatremia. Thiazides, which loserelatively more salt than water, are used occasionally totreat SIADH and can be quite effective.

Free water can be given to attempt to dilute the hyper-natremia. Unfortunately most is wasted by the kidneysbecause of the lack of ADH in the distal tubules. Theadministration of salt, present in hypertonic (choice D)and normal saline (choice E), should be avoided, as thiscan worsen the hypernatremia.

24. The correct answer is B. This patient has severe aplas-tic anemia. This may be from a transient infection, orthe antibiotic she was given may have caused it.Chloramphenicol is very common in third worldnations. Although it is an extremely effective antibiotic(and is used often for nasty gastrointestinal infectionslike typhus), it has a high incidence of aplastic anemia.A bone marrow biopsy, which can help determine thecause of the aplastic anemia, is the next diagnostic stepin this patient’s workup. In the meantime, she needs atransfusion.

Blood and stool cultures (choice A) are not necessary.This patient does not seem to be infected at this time.She has no fever, localizing signs, or evidence in her his-tory that the illness persisted after treatment. For simi-lar reasons, a CT scan of the abdomen (choice C) is notnecessary at this time. If there is an infection in themarrow itself, it will be seen on bone marrow biopsy.

An echocardiogram (choice D) is not necessary. Thispatient’s murmur, consistent with a high velocity flowmurmur, will probably resolve with transfusion.

An erythropoietin level (choice E) can provide indirectevidence of bone marrow function. Cytologic exami-nation (and culture), however, is necessary to deter-mine if this patient has lymphoproliferative disorder,drug-induced toxicity, viral suppression, or marrowreplacement.

25. The correct answer is C. This patient has paroxysmalnocturnal hemoglobinuria (PNH), a cause of hyperco-agulability and hemolytic anemia that is intermittentand difficult to diagnosis. A defect in the hemopoieticcells renders them vulnerable to complement-mediatedlysis, which is precipitated by even a mild acidosis, suchas the mild respiratory acidosis that may occur duringsleep. The increased activation of complement also isaccompanied by platelet aggregation, enhanced expres-sion of tissue factor, and impaired fibrinolysis, all ofwhich place these patients at risk for recurrent clots,particularly in the hepatic and intra-abdominal organs.The presence of PNH should be suspected in anypatient with unexplained or intermittent hemolyticanemia, particularly if accompanied by recurrentvenous thrombosis and pancytopenia. An appropriatediagnostic test is a Ham’s acidification test. The redblood cells are exposed to an acidic solution that acti-vates complement. Abnormal cells lyse, whereas normalones do not.

Although the anemia and leukopenia seen in thesepatients are complement mediated, complement levelsare not a useful test. Unless there is significant, ongoinghemolysis, they will not be markedly abnormal.Further, the measurement of complement levels(choice A) is a nonspecific test, unlike a Ham test.

Gel electrophoresis (choice B) can be used to look forabnormal cell membrane proteins and sometimes is usedto confirm conditions such as hereditary spherocytosis.This technique is not used commonly to diagnosis PNH.

A lactate dehydrogenase level (choice D) can providefurther evidence of hemolysis if elevated but does notassist in the diagnosis of this patient’s condition.

Internal Medicine Answers and Explanations

Page 40: Internal Medicine With Answers

An osmotic fragility test (choice E) is used to test forhereditary spherocytosis. The abnormal morphology isseen easily on a peripheral smear, not seen in this case.The abnormal cells, which have a reduced surface arearelative to volume, lyse when placed in a hypotonic fluid.

26. The correct answer is D. This patient has hereditaryspherocytosis, the most common hemolytic anemiacaused by a red blood cell membrane defect. An elevat-ed MCHC and RDW combined with the finding ofspherocytes are almost pathognomonic for the condi-tion. A positive osmotic fragility test further solidifiesthe diagnosis. In this test, the spherocytes, with analready reduced surface area to volume ratio, lyse whenplaced in a hypotonic solution. Patients often present inchildhood, but mild or intermittent cases may bepicked up in young adults. The definitive treatment issplenectomy.

Hereditary spherocytosis is caused by a genetic defectresulting in abnormal cell membranes that the spleenthen destroys. It is not a classic autoimmune disease,and thus should not be treated with corticosteroids(choice A).

These patients have an appropriate reticulocytosis andare unlikely to benefit from additional exogenous ery-thropoietin (choice B). Further, the definitive treatmentis to stop the hemolysis, not replace the lost cells. Assuch, splenectomy is the most definitive treatment.

If this patient has a mild case or does not want surgery,then no additional treatment (choice C) is necessary.These patients should take supplemental vitamins, par-ticularly folate, to help with the hemolytic anemia, andmay need transfusions if the anemia becomes symp-tomatic.

Stem cell transplant (choice E) is a dramatic interventionthat this condition does not warrant. Although in theoryit can treat hereditary spherocytosis, patients respondwell to a safer and more cost-efficient splenectomy.

27. The correct answer is A. This patient has a classic pre-sentation of atrioventricular nodal reentrant tachycar-dia (AVNRT). Regardless of whether you recognize theclassic EKG findings (initiation by an atrial prematurebeat; pseudo-R waves representing a buried, late P-wave), you should recognize the treatment for a sta-ble, supraventricular tachycardia. The first interventionshould be a vagal maneuver, such as direct pressure onthe eyes, carotid massage, a Valsalva maneuver, or sud-den immersion in cold water (to name a few). If theseinterventions fail, as they often do, a small bolus ofadenosine, which can be doubled and repeated, is thenext appropriate intervention. Adenosine will block the

AV node temporarily. If the rhythm is supraventricular,the atrial rhythm will be easier to see. If the rhythm isnodal in origin, it can be suppressed, allowing for thenative rhythm to take over.

Amiodarone (choice B) does have nodal blocking prop-erties but is usually not necessary for AVNRT, whichcan be chronically suppressed with selective nodalblocking agents that have a safer side-effect profile.

Amlodipine (choice C) is a calcium channel blockerthat acts mainly on the peripheral vasculature. Ifadenosine fails, a nondihydropyridine calcium channelblocker such as diltiazem or verapamil can be used.

DC cardioversion (choice D) is appropriate if thepatient is unstable. This patient, although uncomfort-able, is maintaining an adequate blood pressure.

Digoxin (choice E) can block the atrioventricular node,mainly through increased vagal activity. Adenosine,with much easier dosing and a much shorter half-life, isa much more appropriate acute therapy. Digoxin isreserved as an option for chronic treatment of AVNRT,and SVT in general.

28. The correct answer is A. This patient has new-onsetcongestive heart failure. Her history is very clear for thetelltale signs of increasing left heart filling pressures:paroxysmal nocturnal dyspnea, orthopnea, decreasedexercise tolerance, and pulmonary congestion. Givenher age and track record of never having been hospital-ized, the correct course of action is to obtain anechocardiogram as an inpatient and then begin a med-ication regimen while in the hospital.

Limiting her activity (choice B) and salt intake (choiceC) are not correct because we fail to acknowledge thepresence of a clear process occurring in this patient.Simply prescribing a diuretic (choice D) assumes thatyou understand that she has CHF, but given our lack ofdiagnostic evidence for this, and with no way for us toevaluate the efficacy of the dose prescribed in alleviat-ing her symptoms, it is unacceptable to send thispatient away with such a regimen.

Referring for an echocardiogram and re-evaluating in 3 months (choice E) is unacceptable, given that theinterval of time between her echocardiogram and initi-ation of therapy could result in this patient incurringsevere morbidity or even death.

29. The correct answer is A. Niacin deficiency can causepellagra, which presents with dermatitis, dementia, anddiarrhea, among other symptoms. The dermatitis isclassically in sun-exposed areas and may be mistakenfor a sunburn. It is extremely rare in developed coun-tries, where it tends to occur mainly among alcoholics.

38

USMLE Step 2 Assessment Exam

Page 41: Internal Medicine With Answers

39

In less developed countries, where diets may consist ofmainly low-niacin-content foods (cereals, corn, andsorghum), it is still endemic. Common findings includesymmetric hyperpigmentation, a red tongue, diarrheaand vomiting, and neurologic symptoms, includinganxiety, disorientation, dementia, and encephalopathy.

Thiamine deficiency (choice B) is associated withberiberi and Wernicke-Korsakoff syndrome. Beriberimay be “dry” or “wet.” Dry beriberi refers to neurologicdeficits due to chronic thiamine deficiency, commonlya symmetric peripheral neuropathy. Wet beriberiincludes these symptoms as well as cardiomyopathy.Although this patient may benefit from some thiamine,its deficiency does not explain his symptoms, which aremost likely attributable to niacin deficiency.

Findings associated with a deficiency of pyridoxine,commonly known as vitamin B6 (choice C), includeperipheral neuropathies, seizures, dermatitis, and glos-sitis. Because pyridoxal phosphate is involved in hemesynthesis, a microcytic anemia is also seen.

Vitamin B12 (choice D), or cobalamin, deficiency cancause a macrocytic anemia and neurologic changes, andis classically associated with pernicious anemia. DietaryB12 deficiency is rare and is more likely to be present inthe elderly or those on fad diets. Although this patientmay well benefit from any vitamin supplement, niacin isthe most appropriate supplement to treat his symptoms.

Scurvy, caused by a deficiency of vitamin C (choice E),does not explain this man’s symptoms. Without adequatevitamin C, dysfunctional collagen results in capillaryfragility, poor wound healing, and abnormal hair devel-opment. Patients will often be subject to gum disease andmay lose teeth as the collagen mesh of their gums breaksdown. Ecchymosis and hematomas are often present.

30. The correct answer is A. Influenza vaccine is made frominactivated virus and is safe for pregnant patients. Giventhat pregnant patients are at risk for particularly morbidcases of influenza, all patients without other contraindi-cations should be vaccinated during the second or thirdtrimester. Patients with significant comorbidities shouldbe vaccinated immediately, regardless of trimester.

Oseltamivir (choice B) and amantadine (choice C) areprobably safe in pregnancy (Class C FDA PregnancyRating) but are less effective, more expensive, and not aswell tolerated as a vaccination. This patient would ben-efit more from an immunization at this clinic visit thanfrom these medications.

There is no compelling reason to discourage breast-feeding (choice D) or to delay the immunization untilthe postpartum period (choice E). The vaccine is made

from killed viruses and is not considered transmissible.Further, given the risks of influenza during pregnancy,where it can result in a high rate of hospitalizations,there is no reason to delay immunization. The idealtime to immunize is in October and November, to allowtime for the body to build antibodies before flu season.

31. The correct answer is C. This patient may have primaryHIV infection, which can present weeks to months afterexposure and presents as an acute viral illness. A highdegree of suspicion is needed to make the diagnosis,inasmuch as the signs and symptoms are not specific.Diffuse (as opposed to regional) lymphadenopathy ispresent in 40 to 70% of patients. Mucocutaneous ulcersare further suggestive of the diagnosis, as is the timecourse of the illness. This primary infection correlateswith high degrees of viremia (so the appropriate test is todetermine the HIV RNA level), although the body maynot have had time to produce adequate amounts of anti-bodies for traditional screening tests such as ELISA(choice B) and the confirmatory Western blot (choice E).

A CD4+ T-cell count (choice A) is used to monitor dis-ease progression but not to diagnose the disease.

P24 antigen serologies (choice D) have a sensitivity of 75to 95% and were the early-infection test of choice beforethe widespread availability of polymerase chain reaction-based RNA tests. Given that the viral load is exceedinglyhigh during primary HIV infection (often greater than100,000 copies per milliliter), HIV RNA levels are themost sensitive diagnostic test for primary HIV.

32. The correct answer is C. When evaluating an elevatedalkaline phosphatase level, it is important to determineif it is due to cholestatic liver disease or to bone disease.Alkaline phosphatase is present in a variety of tissues,including bone, liver, intestine, and prostate, but isusually significantly elevated only in bone and liverdisease. Some laboratories have the ability to checkliver specific isoenzymes of alkaline phosphatase; thealternative is to check another marker of cholestaticliver disease that closely correlates with it. Gamma-glu-tamyl transpeptidase (GGT) is elevated in cholestaticliver disease and may be even more sensitive than alka-line phosphatase. If the GGT is normal, it is reasonableto infer that the elevated alkaline phosphatase is frombone. This patient, who is basically asymptomatic, mayhave Paget disease, a condition of increased boneturnover that may present with increased risk of frac-tures, hearing loss and other cranial nerve involve-ment, and an increased risk of osteosarcoma, or simplyabnormal radiographic findings of sclerosis. Given thispatient’s likely diagnosis, there is no need to repeat thealkaline phosphatase test (choice E); there is no reason

Internal Medicine Answers and Explanations

Page 42: Internal Medicine With Answers

to suspect laboratory error or a fluctuating enzymelevel.

Tests of hepatocellular injury, such as alanine amino-transferase (choice A) and aspartate aminotransferase(choice B), are not always elevated in cholestatic diseaseand are thus ideal markers to distinguish the source ofan elevated alkaline phosphatase.

Lactate dehydrogenase (choice D) is present in tissuethroughout the body, including the liver, and is proba-bly most useful as a marker for general cell death andhemolysis and for myocardial infarction in labs notequipped to measure troponins and fractionated creati-nine kinase. It can be elevated in cases of hepatocellulardamage.

33. The correct answer is C. Status epilepticus can be daunt-ing to manage but requires the same basic steps in man-aging any emergency. First, secure the airway. Patientswho do not appear to have major airway concerns canoften get by with a nasal airway or even a simple nasalcannula. This patient, however, has had a prolongedseizure of unknown length and has secretions and bloodin his oropharynx. The airway needs to be managed.

Medications should be started as soon as the airway isstabilized. The first-line treatment is intravenouslorazepam, which has excellent central activity and a longactive half-life in the CNS. Other medications includediazepam, although oral (choice A) is usually not anoption for status epilepticus patients; fosphenytoin; rec-tal diazepam (choice E); and, for resistant seizures, phe-nobarbital (choice D), midazolam, and propofol.

EEG monitoring (choice B) should be started as soon aspossible but is not necessary to guide early manage-ment of status epilepticus.

34. The correct answer is B. The first treatment for symp-tomatic bradycardia is atropine. Peri-infarct patientshave a high degree of counter-regulatory vagal tone,which can result in hypotension and bradycardia, andwill likely respond to atropine. If there is no response toatropine, then transcutaneous (choice D) or transve-nous (choice E) pacing is appropriate. In general,transvenous pacing, which requires less energy to bedelivered and is significantly more comfortable, is thepreferred pacing modality. However, a transcutaneouspacer can be placed quickly and is more useful as anacute treatment.

An emergency revascularization procedure, such ascoronary artery bypass grafting (choice A), is not nec-essary until less invasive procedures have failed.

Phenylephrine (choice C) is not an appropriate agentfor this patient. It results in vasoconstriction without

any beta-mediated increase in heart rate. The increasedperipheral resistance can be expected to reflexivelylower, not increase, this patient’s heart rate.

35. The correct answer is C. This patient has glucose-6-phosphate dehydrogenase (G6PD) deficiency, a criticalenzyme in the hexose monophosphate shunt. The hexose monophosphate shunt serves to protecthemoglobin from oxidative stress. As such, patients areat risk for hemolysis from oxidative damage, which canbe induced by infection, fava beans, physical stress, anda variety of drugs, of which sulfa is notorious.

Abnormal cellular membrane proteins (choice A) cancause hemolytic anemia. Well-described conditionscaused by abnormal cell membranes include paroxysmalnocturnal hemoglobinuria and hereditary spherocyto-sis. This patient’s erythrocytes lack an enzyme but donot have grossly abnormal cellular membranes.

Medications can induce hemolysis through a variety ofmechanisms. First, the drug can induce autoantibodies(choice B), such as in drug-induced lupus. Second, thedrug can bind with cell membranes, creating a newhybrid antigenic target, or hapten. This hapten complexis recognized as foreign and is destroyed by the immunesystem as part of a hapten-mediated cellular destruction(choice D). This mechanism is seen with some penicillinallergies. This patient has a classic presentation forG6PD deficiency, which should be suspected first.

There is no reason to suspect an inheritedhemoglobinopathy (choice E) in this patient.Hemoglobinopathies, such as sickle cell disease and thethalassemias, usually do not present in young adults fol-lowing the use of sulfa drugs.

36. The correct answer is C. Inhaled anticholinergics, par-ticularly ipratropium, are the first line treatment ofCOPD, together with smoking cessation and pul-monary rehabilitation. COPD patients display highparasympathetic tone, which is particularly amenableto muscarinic receptor blocking agents. As ipratropiumis a quaternary ammonium compound and thuscharged (compared with tertiary agents such asatropine), it dissolves freely in water but is extremelylipophilic. This renders it unable to cross the bronchialbarrier, resulting in few systemic side effects.

Albuterol (choice A) is another agent available for thetreatment of COPD. In head-to-head trials, ipratropi-um has been shown to be more effective than albuterolin treating COPD. Note that this is the opposite case inthe treatment of asthma. COPD patients generally suf-fer more side effects (partially because they requirehigher doses) from albuterol than ipratropium. In addi-tion to tachycardia, albuterol can cause pulmonary

40

USMLE Step 2 Assessment Exam

Page 43: Internal Medicine With Answers

41

vasodilatation that may worsen ventilation/perfusionmismatching and result in a slight decrease in arterialoxygen saturation. If patients have significant improve-ment with ipratropium, it should be used as the soleagent. If patients need further treatment, then albuterolmay be an appropriate additional agent.

Steroid inhalers such as flunisolide (choice B) areworthless in the vast majority of patients with COPD.The neutrophilic inflammation of COPD, in contrast tothe eosinophilic inflammation of asthma, does notrespond well to inhaled steroids. If patients have a his-tory of asthma or are extremely ill, a trial of inhaledsteroids may be appropriate. Oral or parenteral steroidssuch as prednisone (choice E), however, are of provenbenefit in the management of an acute exacerbation ofCOPD. Were this patient having a severe exacerbation,prednisone would be an appropriate treatment.

Home oxygen therapy (choice D) is appropriate onlyfor patients with documented low O2 saturation. Theusual criteria is a pulse oximetry saturation of less than88%, either at rest or with exercise, or a PaO2 of lessthan 55% (or less than 59% if comorbid conditions arepresent).

37. The correct answer is C. This patient has an infectiousarthritis. One of the most common causes of infectiousmonoarticular arthritis in young patients is gonorrhea.Symptoms include migratory arthritis, tenosynovitis,and vesiculopustular skin lesions. Diagnosis can bemade through cultures grown on Thayer-Martin medi-um. Treatment for disseminated gonorrhea infectionincludes ceftriaxone.

Acyclovir (choice A) is used in the treatment of herpeticlesions.

Azithromycin (choice B) is used in the treatment ofinfection with chlamydia, which should be consideredin addition to gonorrhea because chlamydia is a com-mon coinfection with gonorrhea.

Although ceftriaxone is the preferred agent,ciprofloxacin (choice D) is an alternative regimen thatmay be used in the treatment of gonorrhea if ceftriax-one cannot be used.

A gram-negative agent, Neisseria gonorrhea does notrespond well to vancomycin (choice E).

38. The correct answer is D. Given this patient’simmunosuppressant medications and clinical presen-tation, tuberculosis is the most likely clinical diagno-sis. The acid-fast bacilli in the induced sputum furthersupport the diagnosis. Isoniazid, rifampin, pyrazi-namide, and ethambutol should be given to thispatient for effective management. Given the high

prevalence of multidrug-resistant tuberculosis world-wide, ethambutol also should be administered toachieve the maximum bacterial kill rate in the first 2months. If the strains are sensitive to the medicationsadministered, ethambutol can be discontinued.

Isoniazid (choice A) alone is not given for the treatmentof active tuberculosis. It is administered typically byitself for prophylaxis.

Isoniazid and rifampin (choice B) may be better thanindividual therapy. As mentioned, however, prevalenceof multidrug-resistant tuberculosis makes it imperativeto use the strongest and most effective combination ofmedications initially.

Isoniazid, rifampin, and pyrazinamide (choice C) formthe backbone of any first-line therapy of tuberculosis.Given the prevalence of multidrug resistant tuberculo-sis, however, ethambutol also should be administeredinitially.

Rifampin (choice E) alone is not suitable treatment

39. The correct answer is E. The patient most likely has thesyndrome of inappropriate secretion of antidiuretichormone (SIADH), in which he is in a sodium-wastingstate and is retaining free water. A pulmonary processcan induce SIADH. The patient should be placed onfree water restriction until the sodium level normalizesgradually. The diagnosis may be confirmed with simul-taneous measurement of plasma and urine osmolarity.

Administering fluid (choice A) may worsen thehyponatremia by effectively inducing the kidneys toretain free water while wasting sodium.

A loop diuretic (choice B) would cause sodium wastingand worsen the hyponatremia.

Similarly, salt tablets (choice C) would not correctSIADH.

The patient is being treated with antibiotics currentlyand he has not necessarily failed treatment (choice D).

40. The correct answer is E. Patients with Wolff-Parkinson-White (WPW) syndrome have an accessory pathway (thebundle of Kent) that can allow rapid conduction of atrialimpulses to the ventricle. The up-sloping R wave istermed a delta wave and represents pre-excitation of theventricle. This accessory pathway from the atria to theventricle also can conduct atrial fibrillation impulsesrapidly, resulting in a very rapid tachycardia. Theincreased risk for sudden death in WPW patients isbelieved to be secondary to rapid ventricular rates duringatrial tachycardia. Understandably, agents are needed thatdirectly suppress the extranodal accessory pathway, ratherthan just block the AV node, a strategy that is appropriate

Internal Medicine Answers and Explanations

Page 44: Internal Medicine With Answers

for most forms of atrial fibrillation. Indeed, there is con-cern that blocking the AV node may increase conductionthrough the more rapidly conducting bundle of Kent,worsening the rapid ventricular response. Procainamide,flecainide, and propafenone are all reasonable choices tosuppress tissue in the accessory pathway.

Nodal blocking agents, such as digoxin (choice A),intravenous diltiazem (choice C), or metoprolol(choice D), are not appropriate choices, as they actmainly on the AV node.

Emergent cardioversion (choice B) is not necessary if apatient is hemodynamically stable. Because cardiover-sion carries its own risks and morbidity, it should notbe a first-line treatment in a stable patient. If thispatient were unstable, then cardioversion would bewarranted. After this patient is in a normal sinusrhythm, or at least is rate controlled, radiofrequencyablation of the accessory bundle should be considered.

41. The correct answer is B. This patient has Wilson disease,a defect in biliary copper excretion that results in copperaccumulation and toxicity in a variety of organs. Wilsondisease should be considered in any young individualwith new-onset psychiatric and neurologic dysfunction.The initial presentation is often insidious, and patients arecommonly misdiagnosed with ADHD when neuropsy-chiatric symptoms predominate, or with autoimmuneliver disease when hepatic dysfunction is the presentingpicture. The most common initial neurologic signs areusually behavioral changes, dysarthria, and dysphagia.Psychosis and depression are common behavioralchanges. It is notable that this patient has a Kayser-Fleisher ring, a ring around the iris of dense brown orgreen copper deposition that is a pathognomonic sign ofWilson disease. The massive, total-body copper overloadresults in increased urinary excretion of copper.Laboratory diagnosis involves finding increased urinarycopper along with decreased ceruloplasmin. Equivocaltests can be clarified with a liver biopsy, which shouldshow increased amounts of cellular copper.

Total serum copper concentration (choice A) is usuallyreduced because there is a deficiency of ceruloplasmin.However, the free copper not bound to the circulatingcopper binding protein is increased, not decreased(choice D).

Ceruloplasmin levels are decreased, not increased(choice C). The genetic defect responsible for Wilsondisease results in a failure of hepatic incorporation ofcopper into apoceruloplasmin, the combination ofwhich forms ceruloplasmin. Low ceruloplasmin levelsare an expected finding in Wilson disease, though thistest is not specific for the condition.

Hepatic concentrations of copper are markedlyincreased, not reduced (choice E). As copper overwhelmsthe liver, which is unable to excrete the substance, cellu-lar damage and eventual necrosis occurs. Free copper is“leaked” into the bloodstream and eventually deposits inother organs.

42. The correct answer is A. This patient has osteoarthritis,which is the most common joint disease. Osteoarthritis,unlike rheumatoid arthritis, is not an inflammatory dis-ease. The initial therapy should include weight loss,exercise, and acetaminophen.

Joint arthroplasty (choice B) is typically reserved forcases in which aggressive medical treatment has beenunsatisfactory, especially if the patient’s quality of lifehas been decreased.

Methotrexate (choice C) and prednisone (choice D) maybe used in the management of rheumatoid arthritis, notosteoarthritis.

Analgesics are typically needed to relieve the pain(choice E).

43. The correct answer is E. This patient is a high-riskpatient who almost certainly has coronary artery diseaseand should be treated as such. Noninvasive testing is notrequired to determine the treatment goals of thispatient. Aggressive risk reduction with a focus on tightglucose control, meeting tight lipid goals (LDL choles-terol level of less than 100 mg/dL), reinforcing smokingcessation, and so on are all appropriate, regardless ofwhat may show up on noninvasive testing. In general,low-risk and high-risk asymptomatic patients do notrequire testing to determine their risk for coronaryartery disease. For low-risk patients, the positive predic-tive value of a test is too low, whereas high-risk patientswarrant treatment regardless of the results of the test.

Coronary computed tomography (choice A) is a rela-tively new technology that looks for calcium deposits inthe coronary arteries. The area and density of calciumare used to generate a coronary calcium score that cor-relates with the extent of atherosclerosis, though notnecessarily the patient’s prognosis. There are no clearguidelines on the use of this modality. Blue Cross BlueShield finds the technique experimental and does notrecommend it, whereas the American College ofCardiology and American Heart Association state thatcoronary CT can be used on “selected” intermediaterisk patients. It can be expected that this patient hassome calcium in his arteries; what to do with a quanti-fied version of this information is unclear.

Electrocardiographic exercise testing (choice B) should beused on intermediate risk patients to further risk-stratify

42

USMLE Step 2 Assessment Exam

Page 45: Internal Medicine With Answers

43

them. Unfortunately, despite its wide use, EKG-stress test-ing has a sensitivity that ranges from 25 to 75% in asymp-tomatic populations. Given this patient’s numerous riskfactors, including baseline EKG abnormalities, this test isunlikely to provide much additional useful information.This patient should be treated as if he has coronary arterydisease. Exercise-stress echocardiography (choice C) andmyocardial perfusion scintigraphy (choice D) have ahigher sensitivity but are expensive, not always available,and have been studied mainly in symptomatic patients.Were this patient to have cardiac symptoms (which he hasa high likelihood of having in the near future), thesewould be excellent tests to determine his near-term riskfor a major event and to determine if invasive testing andtreatment (i.e., cardiac catheterization) are appropriate.At this point, however, further testing is not necessary todetermine this patient’s cardiac risk and treatment goals.

44. The correct answer is B. This patient has an empyema,a complicated parapneumonic effusion. Given the evi-dence of gross pus and concerning findings on pleuralfluid analysis (a low pH and glucose), this patient needsdrainage. The most effective treatment is by way ofchest tube, as repeat thoracenteses (choice E) are rarelyeffective if loculations (evidenced by the lack of layeringon decubitus films) are present.

Broad-spectrum antibiotics (choice A) alone areunlikely to resolve this infection. Likewise other antibi-otics such as ciprofloxacin (choice C) are unlikely toprovide much benefit without definitive drainage.

Pleurectomy (choice D) is a dramatic intervention thatinvolves removal of one or more of the ribs and strip-ping of the pleura. It is rarely used nowadays and is notappropriate for the treatment of the vast majority ofeffusions.

45. The correct answer is C. This patient likely has CNStoxoplasmosis, a common opportunistic infectionoccurring in up to 40% of patients with HIV, and whichis characterized by ring-enhancing lesions on CT scan.The treatment of choice is pyrimethamine, sulfadiazineand leucovorin. Clindamycin is used for patients withsulfa allergies. In addition to imaging with CT or MRI,antibody titers can be used to make the diagnosis. Morethan 15% of patients have negative titers, however, pre-sumably because of B-cell dysfunction in advancedHIV infections.

Amphotericin B (choice A) is appropriate for fungalinfections but not for toxoplasmosis. Toxoplasmosis isan obligate intracellular parasite and does not respondto antifungal agents.

There is always a debate as to whether to give glucocor-ticoids (choice B) when patients have mass lesions. Ingeneral, steroids should not be given to patients withlikely toxoplasmosis. If a B-cell lymphoma is present,also common in this population and that sometimes hasa similar imaging appearance, it will (temporarily)shrink in response to the steroids, confusing the diagno-sis. If the diagnosis is unclear and the patient’s situationdire, a trial of steroids while the diagnosis is confirmedmay be appropriate.

Surgical resection (choice D) is not used for toxoplas-mosis, and consultation is not appropriate at this time.

Trimethoprim/sulfamethoxazole (choice E) is not usedfor toxoplasmosis. It is, however, the main treatment forPneumocystis carinii pneumonia, a common oppor-tunistic infection in this population of patients.

46. The correct answer is C. There are few interventionsthat prolong life and preserve lung function amongCOPD patients. Long-term home oxygen can improvesymptoms of COPD and can prolong life in patientswith severe COPD. Among those with earlier manifes-tations of the disease, the only truly effective treatmentis smoking cessation. Shortly after smoking cessation,the rate of lung decline returns to that of a nonsmoker.If patients continue to smoke, they annually lose two tothree times the lung capacity of nonsmokers, a declinein FEV1 of approximately 50 to 60 mL per year.

Inhaled beta-agonists (choice A) and inhaled choliner-gic antagonists (choice E) are a cornerstone of treatmentfor severe COPD. Cholinergic antagonists are moreeffective in relieving symptoms of COPD, but as the dis-ease progresses, patients often require both forms oftreatment. Although both improve symptoms, neitherpreserves lung function, improves survival, or in anyway alters the course of the disease.

Inhaled glucocorticoids (choice B) are not effective forthe vast majority of COPD patients. For ill patients withan exacerbation of COPD, oral steroids can improvesymptoms and reduce the length of exacerbation.

Lung reduction surgery and lung transplantation(choice D) are treatments for patients with end-stageCOPD. Both interventions improve quality of life, butneither provides a mortality benefit.

47. The correct answer is E. Weakness and fatigue have abroad differential, but myasthenia gravis, particularly ina young woman with ocular and bulbar weakness, needsto be on the differential diagnosis list. The Tensilon test,which involves administering an intravenous dose ofedrophonium and watching for improvements instrength, neatly elucidates the pathophysiology of the

Internal Medicine Answers and Explanations

Page 46: Internal Medicine With Answers

disease. Myasthenia is caused by an antibody-mediatedautoimmune attack directed against nicotinic acetyl-choline receptors. The antibodies initially competitivelyblock the receptor, which is absorbed eventually byendocytosis and destroyed in a lysosome. An increase inacetylcholine can overcome the competitive inhibitionand result clinically in an increase in strength.Edrophonium is a short-acting cholinesterase inhibitorthat results in an increased amount of acetylcholineavailable at the neuromuscular junction. The transientimprovement in strength is clinically apparent, withpatients suddenly being able to tighten their grip or tohold their upward gaze. This should be seen immediate-ly and from the first dose. If there is no immediate effectfrom the first dose of edrophonium (choice B), a differ-ent diagnosis should be suspected.

The medication has a short half-life with a quick onsetof action; gradual improvement (choice A) is unlikely.Long-acting cholinesterase inhibitors are available,though they are not usually the mainstay of therapy.

The patient is expected to have an improvement ofsymptoms with edrophonium. If there is a reproduc-tion of severe symptoms of weakness (choice C), thenthe patient is suffering from a cholinergic crisis inwhich an excess of acetylcholine results in a depolariz-ing neuromuscular blockade. This can occur in patientswho are over treated for their myasthenia, in which casethe Tensilon test is useful for distinguishing an exacer-bation from a cholinergic crisis.

Edrophonium indirectly increases parasympathetictone. A temporary increase in pulse and sympathetictone (choice D) is unlikely.

48. The correct answer is A. Cirrhosis and the resultingportal hypertension often result in gastrointestinalbleeds. Beta-blockers, such as propranolol, are excellentprophylactic therapy for portal hypertension.Nonselective beta-blockers block mesenteric arterydilatation, resulting in unopposed alpha-mediated con-striction. This reduced portal inflow, combined with asmall, direct reduction in portal venous pressure,reduces the potential for portal hypertension-relatedgastrointestinal bleeds. In general, portal hypertensioncan cause gastric or esophageal varices that may pro-duce dramatic, brisk bleeds. Additionally, small ectaticgastric veins can rupture or cause mucosal ischemia,creating less dramatic, slower bleeds. This condition istermed portal hypertensive, or congestive, gastropathy.The only proven treatment is beta-blockade, which isbelieved to be significantly better than no additionaltreatment (choice C).

As the etiology of this patient’s bleed is from vascularirregularities, acid-reducing treatment with H2 blockers(choice B) or proton pump inhibitors (choice D) isunlikely to be effective. Treatments with thermal coag-ulation, injection therapy, acid-reducing treatment,sucralfate, and surgical resection (choice E) have notproven effective in preventing rebleeding.

49. The correct answer is E. It is important to recognize thatthis patient has had a splenectomy, a definitive treatmentfor the anemia and hyperbilirubinemia associated withhereditary spherocytosis. Mild cases are treated with sup-plemental folic acid (choice C) and occasional transfu-sions. Splenectomy does not treat the spherocytes (whichare still present post surgery), but treats the hemolyticanemia that results from splenic destruction of theseatypical cells. There are numerous clues that this patientis asplenic, including most obviously a surgical scar, butalso a normal hemoglobin level and the presence ofHowell-Jolly bodies on the peripheral blood smear. It isimportant to make sure this patient has had a pneumo-coccal vaccine in the past, as splenectomized patients areat risk for overwhelming sepsis. Further, it sounds likethis patient may have had his operation before the cur-rent multivalent formulation was available (whichbecame widespread in the early 1980s). If it is unclear ifthe patient has had a pneumococcal vaccine, he shouldreceive one now. If it can be established with certaintythat the patient had a vaccination, then no additionaltreatment (choice D) is necessary.

Corticosteroids (choice A) play no role in the treatmentof spherocytosis. This is a hereditary defect in the cellwall membrane, not an autoimmune phenomenon.

Dental antibiotic prophylaxis (choice B) is necessaryfor patients with valve abnormalities, not splenec-tomies. Current recommendations for splenectomizedpatients are to not give prophylactic antibiotics beforedental procedures unless there is another reason war-ranting antibiotics.

50. The correct answer is D. Medications need to be startedcautiously and followed closely in elderly patients.Antihypertensives often are started in elderly patientswith systolic hypertension, who, although benefitingimmensely from treatment, are also highly susceptible todrug side effects. Hydrochlorothiazide can cause ahypokalemia that manifests as weakness and musclecramps, or a hyponatremia that presents as lethargy,confusion, and weakness. Additional side effects includeincreased uric acid, increased LDL, and hyperglycemia.

Captopril (choice A) is an angiotensin-convertingenzyme (ACE) inhibitor. Major side effects include hyper-kalemia, chronic cough, and acute renal insufficiency.

44

USMLE Step 2 Assessment Exam

Page 47: Internal Medicine With Answers

45

Clonidine (choice B) is a centrally acting antihyperten-sive known to cause drowsiness, depression, and drymouth. Although it can cause fatigue, it is unlikely tocause muscle cramps. Adverse events that do notrequire a gradual depletion of electrolytes generallyoccur shortly after starting treatment.

Felodipine (choice C) is a calcium channel blocker thatis often used to treat systolic hypertension. Its mostcommon side effect is constipation, though in highdoses it can cause confusion, drowsiness, hypoglycemia,and cardiovascular collapse.

Metoprolol (choice E) and other beta-blockers do notcommonly result in electrolyte abnormalities, the mostlikely explanation for this patient’s symptoms. Majoradverse effects include a low heart rate and symptomsof orthostasis, bronchospastic airways, acute exacerba-tion of heart failure, and facilitation of depression,fatigue, and sexual dysfunction.

51. The correct answer is B. This patient has silica dustexposure and has a clinical picture consistent with sili-cosis and progressive, massive fibrosis. Patients whowork in quarries or around industrial stone grinding,sandblasting, mining, and the like are at risk for pneu-moconiosis caused by fine particles of silica. Patientswith continued exposure, such as this man who hasworked in the family business, will have progressivefibrosis, which shows on radiograph as multiple nod-ules with larger nodules occurring in the apical aspectsof the lungs. Pathologic examination reveals a fibrotic,“honeycombed” lung.

There is no reason to suspect a chronic, postobstructivepneumonia (choice A). A more significant productivecough, evidence of consolidation on exam or radio-graph, or fevers might suggest the presence of infection,none of which were present.

Silicosis does place the patient at an increased risk fortuberculosis. Again, you would expect more in the wayof associated symptoms. Severe coughing, purulentsputum, hemoptysis, and cavitating lesions would bemore suggestive of tuberculoma, in which case youmight find masses composed of acid-fast organismsand necrotic tissue (choice C).

There is no reason to think this patient has small-cell lungcancer (choice D). Although he is a smoker, the bilaterallung findings are consistent with an interstitial diseaserather than a rapidly spreading small cell malignancy.

Asbestos exposure is the major risk factor for mesothe-lioma, which can present as a pleural-based tumor withencroachment into lung parenchyma (choice E). Thispatient’s clinical presentation, exposure history, and

chest radiograph are more consistent with interstitiallung disease such as silicosis.

52. The correct answer is C. This patient likely has cryoglob-ulinemia, a well-described extrahepatic manifestation ofhepatitis C. These abnormal antibodies that precipitateand induce a vasculitis typically manifest as palpable pur-pura and renal involvement, as seen in this patient.Cryoglobulinemia can be severe, necessitating combinedliver and renal transplant in some cases. Given the strongassociation between hepatitis C and this form of vasculi-tis, patients with an unexplained vasculitis, particularly ifmanifesting as palpable purpura, should be tested forhepatitis C.

Hepatitis A (choice A) is an acute, self-limiting diseaseand is not associated with long-term systemic condi-tions. It is not associated with any common forms ofvasculitis.

Hepatitis B (choice B) can cause polyarteritis nodosa, asmall and medium vessel vasculitis. In general, this vas-culitis is less likely to present with palpable purpura andmore likely to have diffuse, systemic involvement thancryoglobulinemia. Again, palpable purpura in a patientwith liver disease is highly suggestive of hepatitis C.

Hepatitis D (choice D) can cause fulminant hepaticfailure when it occurs as a coinfection with hepatitis B.It does not cause these symptoms of vasculitis.

Hepatitis E (choice E) is rare in the United States andgenerally causes an acute, self-limited hepatitis, thoughit may cause fulminant hepatic failure in pregnantwomen.

53. The correct answer is C. This patient likely has autoim-mune thrombocytopenia, also known as idiopathicthrombocytopenic purpura (ITP). The diagnosisrequires excluding other causes of thrombocytopenia,confirming that this is a case of isolated thrombocy-topenia (which dramatically narrows the differentialdiagnosis), and, basically, seeing how the patientresponds to therapy. Given this patient’s young age andthat she is not taking any other medications, it is unlike-ly that she has other potential causes of thrombocy-topenia, such as drug-induced thrombocytopenia ormalignancy. The normal peripheral smear rules outother causes such as platelet clumping (“pseudothrom-bocytopenia”) or microangiopathic conditions. Themost appropriate next step is to give this patient a trialof steroids and see how she responds. Some patientswill respond completely; others will have a response butwill not be able to be successfully tapered from high-dose steroids. These patients may require splenectomy(choice D) and/or intravenous immunoglobulin

Internal Medicine Answers and Explanations

Page 48: Internal Medicine With Answers

(choice E). Patients who still fail therapy may be triedon immune modulating agents such as azathioprine orcyclophosphamide.

Measuring platelet autoantibodies (choice A) seems tomake intuitive sense. Unfortunately, this test is neithersensitive nor specific, and is not required or expected toconfirm the diagnosis of ITP. Currently, the AmericanSociety of Hematology ITP practice guidelines do notrecommend measuring platelet autoantibodies becausethe results can delay and confuse the diagnosis.

A bone marrow biopsy (choice B) is not necessary inmost patients with isolated thrombocytopenia becauseit is extremely rare to have a condition that results inthis isolated abnormality. However, patients over age 60are much more likely to have myelodysplastic syn-drome, which may present like this. As such, patientsover 60 and those with abnormalities on the bloodsmear suggestive of hematoproliferative disordersshould have bone marrow biopsies. This patient, how-ever, does not warrant this invasive procedure.

54. The correct answer is C. Not all cases of syncope requirean extensive workup. If the patient has a clear cause—inthis case a vasovagal episode potentiated by dehydra-tion—further expensive and low-yield diagnostic testsare not necessary. Vasovagal syncope, in which an exag-gerated parasympathetic response causes increasedvagal tone, is the most common cause of syncope and isoften worse in patients with volume depletion. It is notuncommon to feel ill for a short time before and afterthe event; this needs to be differentiated from the pro-drome and postictal confusion that surrounds seizures.

An exercise stress test (choice A) is appropriate ifischemia is suspected. Given that this patient is youngand has no significant past medical history, a stress testis not appropriate for his syncopal episode.

A head CT (choice B) is recommended for all new-onset seizures that present in the emergency room. Thispatient has no evidence of having had a seizure.

A signal-averaged electrocardiogram (choice D) amplifiesand filters the ECG signals in order to detect low-ampli-tude, high-frequency signals in the terminal portion ofthe QRS complex (so-called “late potentials”). These sig-nals are characteristic of subtle ventricular electricalabnormalities that predispose to ventricular tachycardia.The ECG is not an appropriate study in this patient, whois unlikely to have a cardiac cause of his syncope.

Upright tilt testing (choice E) can be used to provoke avasovagal response. It is useful in patients with recurrentsyncopal episodes who do not have evidence of heartdisease or seizures. The results are often not repro-ducible, making this test somewhat difficult to interpret.

55. The correct answer is D. This patient has scurvy,which results from vitamin C deficiency. Collagen syn-thesis depends on adequate stores of vitamin C.Without adequate vitamin C, dysfunctional collagenresults in capillary fragility, poor wound healing, andabnormal hair development. Patients will often be sub-ject to gum disease, and may lose teeth as the collagenmesh of their gums breaks down. The loss of teeth canfurther worsen the poor dietary intake because eatingfruits and vegetables high in vitamin C becomes moredifficult.

Niacin (choice A) deficiency can cause pellagra, whichpresents with dermatitis, dementia, and diarrhea,among other symptoms. It is extremely rare in devel-oped countries, where it tends to occur mainly amongalcoholics. In less developed countries, where diets mayconsist of mainly low-niacin-content foods (cereals,corn, and sorghum), it is still endemic. Common find-ings include symmetric hyperpigmentation, a redtongue, diarrhea and vomiting, and neurologic symp-toms including anxiety, disorientation, dementia, andencephalopathy.

Findings associated with pyridoxine deficiency, com-monly known as vitamin B6 (choice B), includeperipheral neuropathies, seizures, dermatitis, and glos-sitis. Because pyridoxal phosphate is involved in hemesynthesis, a microcytic anemia is also seen. This is arare deficiency but may be seen with drugs that arepyridoxine antagonists, such as isoniazid.

Vitamin B12 (choice C), or cobalamin, deficiency cancause a macrocytic anemia and neurologic changes, andis classically associated with pernicious anemia. DietaryB12 deficiency is rare but is more likely to be present inthe elderly or those on fad diets.

Vitamin K (choice E) is a key vitamin involved in theproduction of liver-dependent clotting factors. While avitamin K deficiency may present as bleeds and anemia,one would expect an elevated prothrombin time fromreduced production of factors II, V, VII, and X.

56. The correct answer is E. This patient has neurosyphilisand needs a significantly higher dose of penicillin forCSF penetration. The treatment of neurosyphilis wouldbe very difficult with the current regimen. Currently, therecommendations are to prescribe 3 to 4 million units ofa short-acting penicillin for a 10- to 14-day course.

Ceftriaxone (choice A) has a 23% failure rate in thetreatment of neurosyphilis.

Doxycycline (choice B) is not used in the treatment ofneurosyphilis. It is used in the treatment of gonorrhea.If used for treating latent syphilis, the duration is for 28days.

46

USMLE Step 2 Assessment Exam

Page 49: Internal Medicine With Answers

47

Penicillin benzathine (choice C) is a long-acting peni-cillin. This dosage is used to treat syphilis that has beenpresent for >1 year.

The treatment should be for not less than 10 days(choice D).

57. The correct answer is D. This patient most likely haspulmonary hypertension from chronic hypoxia. A clueto this diagnosis is the fact that she has been snoring atnight and is moderately obese, thus raising the likeli-hood of sleep apnea. Her chronic hypoxia makes hersusceptible to pulmonary hypertension and an elevatedright ventricular systolic pressure. This diagnosis can bemade with a sleep apnea test or a polysomnographytest.

This patient is denying symptoms that are consistentwith coronary artery disease. She has no chest pain.There is little reason to believe that she would have apositive stress test (choice A).

Similarly, a left heart cardiac catheterization (choice B)would be indicated in a patient with a positive stress testor symptoms consistent with an acute coronary syn-drome.

A lung biopsy (choice C) would be indicated if all otherworkup is negative and interstitial lung disease is sus-pected.

A right heart cardiac catheterization (choice E) wouldconfirm the diagnosis of pulmonary hypertension bymeasuring the pressures in the various cardiac cham-bers. This invasive test can be deferred at this point,however.

58. The correct answer is D. The patient has metastaticspinal cord compression, which occurs in patients withlung, prostate, and breast cancer, as well as with hema-tologic malignancies such as multiple myeloma andlymphoma. Pain accompanies spinal cord compressionin the majority of cases. The site of compression canusually be localized to the site of back or neck pain.Weakness is the second most common symptom. Othersymptoms and findings include overflow incontinence,flaccid paralysis of the lower extremities, absent reflex-es, saddle anesthesia, and impotence. Plain films areuseful in evaluating cord compression due to bonyinvolvement of the spine or compression fracture, butthey cannot show the actual spinal cord itself or any softtissue masses. Magnetic resonance imaging has a veryhigh diagnostic accuracy (95%) in detecting cord com-pression. It can image soft tissue anatomy in greatdetail, and multiple levels of cord impingement can beseen in one exam.

Bone scans (choice A) are more sensitive than plainfilms in detecting metastatic involvement and provideinformation about the entire skeleton in one exam;however, they are not as sensitive and specific in detect-ing spinal metastases as an MRI, and bone scans cannotdelineate soft tissue and spinal cord anatomy. Further,osteolytic metastases produced by multiple myelomaand lymphoma are not detected by bone scan.

A CT scan of the brain (choice B) might possiblydemonstrate a cause for this patient’s weakness, but thetenderness in the spine makes it considerably less likelyto be in the brain than in the cord.

Lumbar puncture (choice C) would rule out carcinoma-tous meningitis, but that is farther down the differentialthan cord compression or brain metastases.

Although checking the patient’s prostate-specific anti-gen (choice E) would be helpful, it would take sometime for the result to return. Of much greater impor-tance would be diagnosing and managing his currentproblem of cord compression, which could result inpermanent loss of neurologic function.

59. The correct answer is B. This patient has a high clinicalsuspicion for a pulmonary embolus. The V/Q scan essen-tially confirms the diagnosis. Oral contraceptive pills inaddition to smoking place her at an increased risk forthromboembolic disease. The most common symptomsof a pulmonary embolism include tachypnea and tachy-cardia. It is also a cause of shock, and a massive pul-monary embolism can result in the loss of blood pressure.This patient has a stable blood pressure at this time. Thus,placing her on heparin immediately would be appropri-ate to prevent the clot burden from increasing.

Aspirin (choice A) would inhibit platelet aggregationbut would not be effective in treating this patient.

Streptokinase (choice C) would be used if the pul-monary embolism were to be lysed immediately. Thiswould be indicated if the patient was hemodynamicallyunstable with a low blood pressure.

Warfarin (choice D) would be appropriate as a long-term choice for anticoagulation in this patient.Ultimately, this patient would be switched to warfarinand taken off heparin. Typically, the duration ofcoumadin would be 6 months, at which time thepatient would be reassessed.

An inferior vena cava filter (choice E) would be indicatedif the patient developed recurrent clots on warfarin.

60. The correct answer is B. Activated charcoal is a fine,nonabsorbable powder that has a network of intercon-necting pores capable of adsorbing different toxins

Internal Medicine Answers and Explanations

Page 50: Internal Medicine With Answers

quickly. It is useful for a variety of substances and shouldbe given in most cases of acute poisoning, as it isextremely rare to be 100% sure of what a patient mayhave ingested. Unfortunately, not all substances are wellabsorbed by charcoal. Small, highly ionized chemicals,such as metals, electrolytes, acids, and alkali are not wellabsorbed. As such, iron overdose or lithium toxicity isnot affected by charcoal. Further, hydrocarbons and alco-hols also are unaffected by charcoal, making it impossi-ble to treat an intoxicated individual with charcoal.

Acetaminophen (choice A), amitriptyline (choice C),aspirin (choice D), and digoxin (choice E) are alladsorbed by the micropores of activated charcoal. In gen-eral, the sooner the administration of charcoal, the greaterthe likelihood of stopping drug absorption. Charcoalshould not be withheld arbitrarily, however, because of apresumed timeline of drug absorption. Many medica-tions come in delayed release forms or have gut dysmotil-ity effects (for example, the anticholinergic effects ofamitriptyline) that can delay systemic absorption.

61. The correct answer is A. Crohn disease, and other dis-eases that cause fat malabsorption such as sprue andpancreatic disease, can result in fat-soluble vitamindeficiency. The fat-soluble vitamins are D, E, A, and K(DEAK). Vitamin A deficiency can result in night-blindness, conjunctival hyperkeratosis (Bitot spots),and a variety of dermatologic manifestations.

Vitamin B (choice B) encompasses a variety of vita-mins, with a host of different deficiencies. These arerarely seen in developed countries because vitamin B isubiquitous in the food supply. Further, this patient isnot at risk for deficiencies in water-soluble vitamins.

Vitamin D (choice C) deficiency manifests as osteo-porosis in adults and rickets in children, as vitamin D isnecessary for effective calcium absorption.

Vitamin E (choice D) deficiency is extremely uncom-mon but may manifest as ataxia and hemolytic anemia.

Vitamin K deficiency (choice E) can occur in newbornsand in patients on total parenteral nutrition or long-term antibiotics (which can alter vitamin K–producinggut flora). A prolonged prothrombin time or frankbleeding may be present.

62. The correct answer is D. What began as simple urinarytract obstruction has become the dreaded urologicemergency of obstruction plus infection. In addition toantibiotics, emergency decompression of the urinarytract above the obstruction is needed.

Antibiotics alone (choice A) will not be enough; anephrostomy is needed.

Alkalinization of the urine (choice B) can dissolve uricacid stones. Uric acid stones are not calcified, so she doesnot have that kind of stone. Furthermore, she is facingan emergency that requires immediate resolution.

Endoscopic removal of the stone (choice C) soundsappealing, but it is absolutely contraindicated.Manipulation and instrumentation of the urinary tractin the present situation would lead to systemic sepsis. Itis too late to remove the stone now. After decompres-sion, it may be okay to do it.

The same could be said for shock wave lithotripsy(choice E). It might be acceptable to do it once the acuteproblem is over, but not right now.

63. The correct answer is C. In the presence of progressive,unstable angina, he should be evaluated for potentialcoronary revascularization before major noncardiacsurgery is performed. If he is a suitable candidate, bothoperations could be performed simultaneously, or hecould undergo the revascularization before hisaneurysm is fixed.

Echocardiograms (choice A) are used to work up sus-pected morphologic abnormalities of the heart. Thatprocedure would not reveal the necessary details aboutthe coronary circulation.

Exercise tolerance test and thallium scan (choice B) areindicated to unmask silent coronary disease in high-risk individuals. It would be hazardous to subject apatient with unstable angina to this particular test.

If he were in congestive failure, digitalis and diuretics(choice D) could be part of his treatment, but he has noevidence of failure at this time.

The standard time interval to wait to reduce operativerisk after an MI is 6 months. This man is already 8months out. Waiting a full year as proposed in choice Eadds no further advantage.

64. The correct answer is C. This patient has acuteGuillain-Barré syndrome (GBS), an acute demyelinat-ing polyradiculopathy that is often associated with arecent infection. The manifestations include ascendingweakness, areflexia, and an increase in cerebrospinalfluid protein. Treatment is typically with plasmaphere-sis or intravenous immunoglobulin.

Glucocorticoids (choice A) and interferon-beta (choiceB) are not effective in the treatment of acute GBS. Theyare often used in multiple sclerosis.

Pyridostigmine (choice D) and thymectomy (choice E)are not effective in the treatment of acute GBS. They aretreatments for myasthenia gravis.

48

USMLE Step 2 Assessment Exam

Page 51: Internal Medicine With Answers

49

65. The correct answer is D. The patient’s electrocardiogramshows that she has myocardial ischemia of the left ventri-cle. The diabetes and hypercholesterolemia are risk fac-tors for her coronary artery disease. Treatment must beinitiated to decrease her myocardial oxygen demand.

Costochondritis (choice A) and musculoskeletal pain(choice B) are not associated with electrocardiographicchanges.

Myocardial infarction (choice C) is typically associatedwith either ST elevations or elevated serum markers,neither of which is mentioned in this vignette.

The electrocardiographic findings are consistent withischemia, not a pulmonary embolism (choice E).

66. The correct answer is C. The most common cause ofmicrocytic anemia is iron deficiency, and a low serumferritin confirms that diagnosis. This man has probablybeen chronically losing blood, and the most likelysource is the gastrointestinal tract. Cancer of the rightside of the colon is a good bet, and once occult blood isfound in the stool, a colonoscopy should follow.

Coombs test (choice A) is used for the diagnosis ofhemolytic anemia, which is normocytic and does notdeplete iron stores.

Hemoglobin electrophoresis (choice B) is used for thediagnosis of thalassemia. The anemia in that disease isindeed microcytic, but the patient would have normaliron stores (normal serum ferritin).

The Schilling test (choice D) has a role when patientshave macrocytic anemia, as it helps differentiate perni-cious anemia from intestinal malabsorptive disorders.

Anemia caused by renal insufficiency is normocytic, inwhich case serum creatinine (choice E) would be a use-ful test.

67. The correct answer is A. The history is suggestive ofepisodes of hypoglycemia, possibly triggered by aninsulinoma. Fasting should induce the symptoms, atwhich time one could demonstrate the presence ofhypoglycemia and high levels of insulin and C peptide.Resolution of the symptoms by glucose administrationwould then complete the diagnostic “Whipple triad.”CT scan of the pancreas would follow.

Tumors involving the frontal lobe may produce behav-ioral changes, and funduscopic examination (choice B)and MRI (choice D) would be helpful to diagnose thatproblem. In that case, though, the behavioral changesshould be accompanied by persistent headaches, visualchanges, and possibly olfactory deficits (Foster Kennedysyndrome). There would be no correlation with fastingor resolution by eating.

Liver failure is a potential cause of hypoglycemia, but thelatter would not occur until liver function is severelycompromised. The patient would have overt signs ofliver failure, rather than a minor, occult deficiency thatwould be discovered by liver function tests (choice C).

If the indicated tests were to show that his symptomsare indeed caused by hypoglycemia, at which timeblood levels of insulin were found to be high while theC peptide was low, self-administration of insulinshould be suspected. It would be at that point that psy-chiatric evaluation (choice E) would be indicated.

68. The correct answer is C. The presentation is highly sug-gestive of Zollinger-Ellison (Z-E) syndrome. Furtherstudies after her serum gastrin level is determined mightinclude secretin stimulation test, if needed for confirmation, and CT scan of the pancreas to locate thegastrinoma.

Secretory diarrhea in this setting is part and parcel ofZ-E syndrome, and pathology in the intestinal mucosaneed not be sought (choice A).

Biopsy of the ulcer (choice B) is dangerous and unnec-essary. Biting into an ulcer that recently bled is an invi-tation to rebleeding, and ascertaining if the ulcer isbenign or malignant is indicated in the case of gastriculcers but is not pertinent for duodenal ulcers.

At some point, serum calcium (choice D) needs to bemeasured. Gastrinomas may coexist with parathyroidadenomas. It would not be the first test needed, however,before the presence of Z-E has been established.

Upper gastrointestinal series with barium (choice E)does not add any information to that already providedby the endoscopy.

69. The correct answer is B. This man has massive gas-trointestinal bleeding, and the nasogastric aspirate indi-cates that it is coming from beyond the ligament ofTreitz. Approximately 70% of patients who bleed froma source distal to that landmark do so from diverticulo-sis. Approximately one quarter of those present withmassive bleeding. The offending diverticula are usuallyin the right colon and thus are not visible to a proc-tosigmoidoscopic examination. The usual patient withthis presentation is old enough to have diverticulosis.All the numbers quoted fit this particular case.

Colon cancer (choice A) also occurs in older people. It isalso a frequent source of bleeding, but typically thebleeding is not massive. Furthermore, the left side of thecolon hosts cancers more frequently than the right. Thisman could be bleeding from a colonic cancer, but obvi-ously that would not be the most likely situation.

Internal Medicine Answers and Explanations

Page 52: Internal Medicine With Answers

Duodenal ulcer (choice C) has been excluded by thenasogastric aspirate. Upper gastrointestinal sourcesalways have to be considered when people bleed perrectum (a common dictum says that a common causefor lower gastrointestinal bleeding is in fact upper gas-trointestinal bleeding), and that is why the nasogastricaspiration is done. Once one sees green fluid withoutblood, however, one stops thinking of upper gastroin-testinal lesions.

Inflammatory bowel disease (choice D) can bleed, butthat source is way down on the list of possibilities.

Portal hypertension (choice E) can produce massivebleeding. Typically, however, it is by way of esophagealvarices (which have been ruled out by the nasogastricaspirate) or hemorrhoids (which have been ruled outby direct examination of the anorectal area).

70. The correct answer is A. Although this patient has theclinical presentation of diverticulitis, the key point isthat the patient was recently treated with gentamicinand is therefore at risk for nephrotoxicity. This takes theform of acute tubular necrosis and may either be olig-uric or nonoliguric. Patients with the nonoliguric formgenerally recover their renal function if the aminogly-coside is discontinued and the patient is adequatelyhydrated.

Diverticulitis (choice B) has no direct effect on renalfunction. Theoretically, patients with diverticulitiswithout septic shock may develop renal insufficiency,but there is no evidence of that scenario here.

Left ureteral obstruction (choice C) may occur in apatient with sigmoid diverticulitis but would not causerenal insufficiency unless the patient also had right kid-ney dysfunction.

Pyelonephritis (choice D) is not described by the phys-ical and CT scan findings in this patient, and unilateralpyelonephritis would not cause renal insufficiency.

There is no evidence by physical examination that thispatient is in shock (choice E).

71. The correct answer is D. This previously healthy youngwoman has significant systolic and diastolic hyperten-sion. The classic clue here is an upper abdominal bruit,which suggests the diagnosis of renovascular hyperten-sion. In a young woman, the diagnosis of renovascularhypertension is usually due to fibromuscular dysplasia,which is usually unilateral and unifocal.

Although aortic dissection (choice A), coarctation of theaorta (choice B), hypertrophic cardiomyopathy (choiceC), and Takayasu arteritis (choice E) are all associatedwith hypertension, they do not produce the upper

abdominal bruit heard here. Furthermore, choices A, B,and E will generally produce a discrepancy in bloodpressure between the arms or between the arms and legs.There is no evidence on exam of a hypertrophic car-diomyopathy, i.e., no murmurs or S4 are described.

72. The correct answer is E. This case describes a typical pre-sentation of bacterial meningitis. Suggestive of meningitisare the patient’s headache and photophobia. Suggestive ofa bacterial cause are the patient’s temperature to 39.5 C(103.1 F), cloudy CSF, and positive Gram stain.Streptococcus pneumoniae is one of the two leading causesof purulent meningitis in this age group. Neisseria menin-gitidis is the other cause and may occur in epidemics.

Encapsulated yeast with hyphae (choice A) is a rarecause of meningitis, certainly in an immunocompetenthost. Cryptococcus neoformans, the leading cause in theyeast family, is an oval, budding yeast that does not existas hyphae.

Gram-negative bacilli in pairs (choice B) describes adiverse group of organisms. Typically, these are found inthe GI tract and respiratory tract. The two that are morecommonly associated with meningitis are Haemophilusinfluenzae and Escherichia coli. These are the leadingcauses in infants and children younger than 6 years old.(Group B strep is most common in neonates.)

Gram-negative cocci in clusters (choice C) is not a typ-ical description of an organism. Neisseria, the mostcommon Gram-negative coccus, is a diplococcus thatoccurs in pairs.

Gram-positive bacilli in pairs (choice D) include fourgenera, of which Listeria monocytogenes is commonlyassociated with meningitis in neonates and immuno-compromised hosts.

73. The correct answer is C. This patient is having an acutemyocardial infarction (MI), as evidenced by ST seg-ment elevations on the EKG in the setting of the pastmedical history. The treatment for acute MI, with eitherST segment elevations of more than 1 mm in two con-tiguous leads or a new left bundle branch block, isimmediate reperfusion with either intravenous throm-bolytics or coronary balloon angioplasty via coronaryartery catheterization. This is the absolute standard ofcare.

Admission to the coronary care unit for exclusion of MI(choice A) is inappropriate because all evidence sug-gests that the patient is actively infarcting.

Emergent coronary artery bypass surgery (choice B) isnot done on an emergent basis, but rather the patientwould be brought to the cardiac catheterization labora-tory for percutaneous intervention.

50

USMLE Step 2 Assessment Exam

Page 53: Internal Medicine With Answers

51

Cardioversion (choice D) is not indicated, inasmuch asthe patient is not having any unstable arrhythmias asso-ciated with the acute MI.

A PET scan (choice E) is not done on an emergent basisbut to detect residual functional myocardium after anischemic event.

74. The correct answer is A. The 5-TU PPD reaction in thispatient is considered to be negative because he has norisk factors, is from a low-incidence group, and has areaction of less than 15 mm. The reaction to 5-TU PPDhas to be 15 mm or greater to be considered a positiveresult in a person with no risk factors who is from alow-incidence group. Therefore, no further manage-ment is needed for this patient.

Combination therapy such as isoniazid, rifampin, andpyrazinamide (choice B) is used for treating activetuberculosis. This patient has no signs or symptoms tosuggest active tuberculosis.

Isoniazid alone (choice C), when used for 6 to 12months as prophylactic therapy in a patient with a pos-itive 5-TU PPD reaction, significantly reduces the riskof tuberculosis.

Rifampin alone (choice D) can be used as an alternativein prophylactic therapy (usually in combination withethambutol) in patients who are potentially infectedwith drug-resistant organisms. There is no role for thesemedications in this patient who has a negative result.

Bronchoscopy (choice E) is used as a diagnostic tool toconfirm active tuberculosis. Because active tuberculosisis not suggested in this case, bronchoscopy has no rolein the care of this patient.

75. The correct answer is C. This patient likely has influenza,though other viral etiologies cannot be ruled out. If arapid influenza test is available (not common in mostclinics), then a “test and treat” strategy is useful.Unfortunately, viral antibody titers (choice A) rise afterthe acute phase of the illness, thus negating their useful-ness in treatment. Most cases of influenza are self-limited.However, if a patient desires treatment, amantadine andthe neuraminidase inhibitors (such as oseltamivir) havebeen proven to reduce the duration of illness by about aday if started within 2 days of symptom onset. If thepatient has been ill for more than 2 days, however, then noadditional treatment is available (choice E).

Empiric macrolide antibiotics (choice B) are not appro-priate for this patient, inasmuch as her illness is probablynot bacterial. Patients with COPD will often have flaresof bronchitis, caused or exacerbated by organisms thatare susceptible to macrolides.

High-dose vitamin C and zinc lozenges (choice D) haveboth been proposed as “cures for the common cold.”Unfortunately, in rigorous, controlled studies neithertreatment has shown benefit. That said, if a patientwishes to take supplemental vitamin C and zinc, thereis little harm to be had and perhaps much to be gainedfrom maximizing the placebo effect of these treatments.

Internal Medicine Answers and Explanations

Page 54: Internal Medicine With Answers

kaplanmedical.comBL3

163A

Prin

ted

in U

SA

©20

03